73
QUESTION 1. Q.1 ) NISARGRUNA technology is related to a) Biomethanation b) Incineration c) Bioremediation d) None of the above Correct Answer: A Your Answer: Unanswered Explanation NISARGRUNA technology based on biodegradable waste resource for biomethanation was developed by Sharad Kale, a professor at the Bhabha Atomic Research Centre at Mumbai QUESTION 2. Consider the following statement about investor-state dispute settlement: 1.The ISDS mechanism permits companies to drag governments to international arbitration without exhausting the local remedies. 2.India has always supported ISDS mechanism Select the correct statement: a) a ) Only 1 b) Only 2 c) Both 1 and 2 d) Neither 1 nor 2 Correct Answer: A Your Answer: A Explanation - ISDS is an instrument of public international law and provisions are contained in a number of bilateral investment treaties, in certain international trade treaties, such as NAFTA, and the proposed TPP and CETA agreements. ISDS is also found in international investment agreements, such as the Energy Charter Treaty. -India has rejected such mechanism. It clearly held that only after all local options have been exhausted for settling disputes between a corporate and a government, then the issues can be taken up in international arbitration tribunals. It also held that such provisions could be a part of bilateral agreements but they cant be allowed in a www.freeupscmaterials.org freeupscmaterials.org

QUESTION 1. Q.1 ) NISARGRUNA technology is related to Your ... · Exam Title : TS03 CA-JUNE 17 L2 Email : [email protected] Contact : 8250416813 Civilsdaily Email: [email protected]

  • Upload
    others

  • View
    2

  • Download
    0

Embed Size (px)

Citation preview

Page 1: QUESTION 1. Q.1 ) NISARGRUNA technology is related to Your ... · Exam Title : TS03 CA-JUNE 17 L2 Email : sandeepbiswas221@gmail.com Contact : 8250416813 Civilsdaily Email: hello@civilsdaily.com

Exam Title : TS03 CA-JUNE 17 L2

Email : [email protected]

Contact : 8250416813

Civilsdaily

Email: [email protected] 0

QUESTION 1.

Q.1 ) NISARGRUNA technology is related to

a) Biomethanation

b) Incineration

c) Bioremediation

d) None of the above

Correct Answer: A

Your Answer: Unanswered

Explanation

NISARGRUNA technology based on biodegradable waste resource for biomethanation was developed by

Sharad Kale, a professor at the Bhabha Atomic Research Centre at Mumbai

QUESTION 2.

Consider the following statement about investor-state dispute settlement:

1.The ISDS mechanism permits companies to drag governments to international arbitration without

exhausting the local remedies.

2.India has always supported ISDS mechanism

Select the correct statement:

a) a ) Only 1

b) Only 2

c) Both 1 and 2

d) Neither 1 nor 2

Correct Answer: A

Your Answer: A

Explanation

- ISDS is an instrument of public international law and provisions are contained in a number of bilateral

investment treaties, in certain international trade treaties, such as NAFTA, and the proposed TPP and

CETA agreements. ISDS is also found in international investment agreements, such as the Energy Charter

Treaty.

-India has rejected such mechanism.

It clearly held that only after all local options have been exhausted for settling disputes between a corporate

and a government, then the issues can be taken up in international arbitration tribunals.

It also held that such provisions could be a part of bilateral agreements but they cant be allowed in a

www.freeupscmaterials.org

freeupscmaterials.org

Page 2: QUESTION 1. Q.1 ) NISARGRUNA technology is related to Your ... · Exam Title : TS03 CA-JUNE 17 L2 Email : sandeepbiswas221@gmail.com Contact : 8250416813 Civilsdaily Email: hello@civilsdaily.com

Exam Title : TS03 CA-JUNE 17 L2

Email : [email protected]

Contact : 8250416813

Civilsdaily

Email: [email protected] 0

multilateral agreement.

Source: http://www.civilsdaily.com/investment-pact-system-needs-review/

QUESTION 3.

Q.3 ) Dhanush Artillery Gun is an upgraded version of

a) Swedish 155-mm Bofors howitzers

b) Russian M-46

c) Pinaka

d) None of the above

Correct Answer: A

Your Answer: A

Explanation

Dhanush is an upgraded version, based on the original design of the Swedish 155-mm Bofors howitzers,

which India procured in the mid-1980s. It is a 155-mm, 45-calibre gun with a maximum range of 40 km in

salvo mode, compared to the 39-calibre, 27-km range of the original guns.

QUESTION 4.

Which of the following statements given below is/are correct in context with the recently launched Electoral

Bonds?

1. They are issued by Scheduled Commercial banks.

2. Electoral Bonds cannot be purchased by paying cash.

3. Electoral bond was announced in the Union Budget 2017-18.

Select the correct option using the codes given below.

a) 1 only

b) 1 and 3 only

c) 1, 2 and 3

d) 2 and 3 only

Correct Answer: C

www.freeupscmaterials.org

freeupscmaterials.org

Page 3: QUESTION 1. Q.1 ) NISARGRUNA technology is related to Your ... · Exam Title : TS03 CA-JUNE 17 L2 Email : sandeepbiswas221@gmail.com Contact : 8250416813 Civilsdaily Email: hello@civilsdaily.com

Exam Title : TS03 CA-JUNE 17 L2

Email : [email protected]

Contact : 8250416813

Civilsdaily

Email: [email protected] 0

Your Answer: B

Explanation

-Electoral Bond is a financial instrument for making donations to political parties.

-These are issued by Scheduled Commercial banks after getting permission from the Central.

-Government to intending donors, but only against cheque and digital payments (it cannot be purchased by

paying cash).

-These bonds shall be redeemable in the designated account of a registered political party within the

prescribed time limit from issuance of bond. Electoral bond was announced in the Union Budget 2017-18.

Source: http://www.civilsdaily.com/op-ed-snap-the-black-box/

QUESTION 5.

Which of the following Insurance schemes for farmers given below has/had provisions for local calamities

such as landslide, hailstorm, inundation, etc ?

1. National Agriculture Insurance Scheme(NAIS)

2. Pradhan Mantri Fasal Bima Yojana

3. Experimental Crop Insurance

Select the correct option using the codes given below.

a) 1 only

b) 2 only

c) 2 and 3 only

d) 1.2 and 3

Correct Answer: B

Your Answer: Unanswered

Explanation

www.freeupscmaterials.org

freeupscmaterials.org

Page 4: QUESTION 1. Q.1 ) NISARGRUNA technology is related to Your ... · Exam Title : TS03 CA-JUNE 17 L2 Email : sandeepbiswas221@gmail.com Contact : 8250416813 Civilsdaily Email: hello@civilsdaily.com

Exam Title : TS03 CA-JUNE 17 L2

Email : [email protected]

Contact : 8250416813

Civilsdaily

Email: [email protected] 0

Pradhan Mantri Fasal Bima Yojana, unlike the previous ones, covers local calamities too, such as

landslide, hailstorm, inundation, etc. inundation was not covered by the previous schemes(1 and 3).

QUESTION 6.

The Purchasing Managers Index (PMI) is based on some major indicators. Which of the following is not

the major indicator?

a) Inventory levels

b) Production

c) Employment environment

d) All of the above are major indicators

Correct Answer: D

Your Answer: D

Explanation

The Purchasing Managers Index (PMI) is an indicator of the economic health of the manufacturing sector.

The PMI is based on five major indicators: new orders, inventory levels, production, supplier deliveries and

the employment environment. The purpose of the PMI is to provide information about current business

conditions to company decision makers, analysts and purchasing managers.

http://www.civilsdaily.com/services-sector-gains-momentum-in-may/

QUESTION 7.

Q.7 ) Consider the following statements regarding the El Nio Southern Oscillation (ENSO):

1. The cool phase of ENSO is called La Nia.

2. Trade Winds become weak in the El Nino year.

3, ENSO water circulation happens between Australia and India.

Which of the following statements given below is/are correct?

a) 1 and 2 only

b) 2 and 3 only

c) All are correct

d) 1 and 3 only

Correct Answer: A

Your Answer: Unanswered

Explanation

www.freeupscmaterials.org

freeupscmaterials.org

Page 5: QUESTION 1. Q.1 ) NISARGRUNA technology is related to Your ... · Exam Title : TS03 CA-JUNE 17 L2 Email : sandeepbiswas221@gmail.com Contact : 8250416813 Civilsdaily Email: hello@civilsdaily.com

Exam Title : TS03 CA-JUNE 17 L2

Email : [email protected]

Contact : 8250416813

Civilsdaily

Email: [email protected] 0

El Nino-Southern Oscillation (ENSO) water circulation happens between Australia and Peru. Two things

become weak: Cold Peru Current ans Trade Winds(opposite for La Nina).

http://www.thehindu.com/todays-paper/tp-national/el-nino-is-weaker-than-anticipated-says-imd/article18733895.ece

QUESTION 8.

Q.8 ) Which of the following is/are not the founding members of the Shanghai Cooperation

Organisation(SCO)?

1. Tajikistan

2. Russia

3. Turkmenistan

4. Kyrgyzstan

Select the correct option using the codes given below.

a) 2 and 3 only

b) 3 only

c) 1, 3 and 4 only

d) 2 only

Correct Answer: B

Your Answer: B

Explanation

The Shanghai Cooperation Organization (SCO), or Shanghai Pact, is a Eurasian political, economic, and

military organisation which was founded in 1996 in Shanghai by the leaders of China, Kazakhstan,

Kyrgyzstan, Russia, Tajikistan, and Uzbekistan. These countries, except for Uzbekistan, had been

members of the Shanghai Five, founded in 1996; after the inclusion of Uzbekistan in 2001, the members

renamed the organisation.

Source: http://www.civilsdaily.com/india-to-get-sco-full-membership-in-astana-say-russia-and-china/

QUESTION 9.

www.freeupscmaterials.org

freeupscmaterials.org

Page 6: QUESTION 1. Q.1 ) NISARGRUNA technology is related to Your ... · Exam Title : TS03 CA-JUNE 17 L2 Email : sandeepbiswas221@gmail.com Contact : 8250416813 Civilsdaily Email: hello@civilsdaily.com

Exam Title : TS03 CA-JUNE 17 L2

Email : [email protected]

Contact : 8250416813

Civilsdaily

Email: [email protected] 0

The Shanghai Cooperation Organization (SCO) covers which of the following aspects of International

relations?

1. Political

2. Economic

3. Millitary

Select the correct options using the codes given below.

a) 1 and 2 only

b) 2 only

c) 3 only

d) 1, 2 and 3

Correct Answer: D

Your Answer: D

Explanation

The Shanghai Cooperation Organization (SCO), or Shanghai Pact, is a Eurasian political, economic, and

military organisation.

source: http://www.civilsdaily.com/op-ed-snap-clouded-coherence/

QUESTION 10.

Q.10) 'GeneXpert' device which was recently seen in the news can be used to diagnose:

a) TB

b) HIV

c) Hepatitis C

d) All of the above

Correct Answer: D

Your Answer: D

Explanation

-The World Health Organization (WHO) has released a new device 'GeneXpert' which can be used for

multi-disease testing. The WHO is recommending use of these state-of-the-art portable machines. These

devices are based on Molecular Technology.

www.freeupscmaterials.org

freeupscmaterials.org

Page 7: QUESTION 1. Q.1 ) NISARGRUNA technology is related to Your ... · Exam Title : TS03 CA-JUNE 17 L2 Email : sandeepbiswas221@gmail.com Contact : 8250416813 Civilsdaily Email: hello@civilsdaily.com

Exam Title : TS03 CA-JUNE 17 L2

Email : [email protected]

Contact : 8250416813

Civilsdaily

Email: [email protected] 0

-The GeneXpert can be used to diagnose TB and HIV infections, and quantitatively measure HIV and

hepatitis C viral loads.

source: http://www.civilsdaily.com/who-for-use-of-devices-to-test-multiple-diseases/

QUESTION 11.

Odisha government has recently developed a real time flood forecasting model on one of its River. This

Model is developed on the river

a) Bhargavi

b) Daya

c) Devi

d) Mahanadi

Correct Answer: D

Your Answer: D

Explanation

It is developed for the entire Mahanadi river basin for better flood control. The model is supported by the

State Climate Change Innovation Programme.

QUESTION 12.

Consider the following statements:

1. The new WPI series does not include any indirect taxes

2. The new methodology has led to consistently lower WPI inflation than earlier series

Which of the above is/are correct?

a) 1 only

b) 2 only

c) Both 1 and 2

d) Neither 1 nor 2

Correct Answer: C

Your Answer: C

Explanation

-The new WPI series does not include any indirect taxes, which brings it conceptually closer to the

producer prices and also removes fiscal impulse impact.

-The new methodology has led to consistently lower WPI inflation than earlier series. For 2016-17, the WPI

www.freeupscmaterials.org

freeupscmaterials.org

Page 8: QUESTION 1. Q.1 ) NISARGRUNA technology is related to Your ... · Exam Title : TS03 CA-JUNE 17 L2 Email : sandeepbiswas221@gmail.com Contact : 8250416813 Civilsdaily Email: hello@civilsdaily.com

Exam Title : TS03 CA-JUNE 17 L2

Email : [email protected]

Contact : 8250416813

Civilsdaily

Email: [email protected] 0

inflation averaged a mere 1.7 per cent in the new series as against 3.7 per cent growth in the old base

series.

QUESTION 13.

Consider the following statements:

1. IIP and WPI base year have been changed to 2015-16

2. Index of Industrial Production (IIP) is released by Central Statistics Office

Which of the above is/are correct?

a) 1 only

b) 2 only

c) Both 1 and 2

d) Neither 1 nor 2

Correct Answer: B

Your Answer: D

Explanation

-On May 12, with an aim to bring major macroeconomic indicators of IIP and WPI in line with national

accounts, the government changed its base year to 2011-12.

-Index of Industrial Production is compiled and published every month by Central Statistics Office (CSO) of

the Ministry of Statistics and Programme Implementation with a time lag of six weeks from the reference

month. i.e., at the time of release of IIP data, quick estimates for the relevant month along with revised and

final indices of previous two months respectively, (on the basis of updated production data) are released.

QUESTION 14.

Recently, Uttarakhand and Haryana have become 4th and 5th states in the country to be open defecation

free. Which was the first state to be declared open defecation free?

a) Sikkim

b) West Bengal

c) Himachal Pradesh

d) Kerala

Correct Answer: A

Your Answer: A

Explanation

www.freeupscmaterials.org

freeupscmaterials.org

Page 9: QUESTION 1. Q.1 ) NISARGRUNA technology is related to Your ... · Exam Title : TS03 CA-JUNE 17 L2 Email : sandeepbiswas221@gmail.com Contact : 8250416813 Civilsdaily Email: hello@civilsdaily.com

Exam Title : TS03 CA-JUNE 17 L2

Email : [email protected]

Contact : 8250416813

Civilsdaily

Email: [email protected] 0

Sikkim was the first state to be declared open defecation free in the country followed by Himachal Pradesh

and Kerala.

source: http://pib.nic.in/newsite/PrintRelease.aspx?relid=165840

QUESTION 15.

Consider the following statements about VAJRA (Visiting Advanced Joint Research) Faculty Scheme

1. It is a dedicated programme exclusively for the resident Indians to performs researches with international

scientists and overseas universities.

2. The scheme will be implemented by the Science and Engineering Research Board (SERB).

Which of the statements given above is/are correct?

a) 1 only

b) 2 only

c) Both 1 and 2

d) Neither 1 nor 2

Correct Answer: B

Your Answer: C

Explanation

-VAJRA (Visiting Advanced Joint Research) Faculty Scheme is a dedicated program exclusively for

overseas scientists and academicians with emphasis on Non-resident Indians (NRI) and Persons of Indian

Origin (PIO) / Overseas Citizen of India (OCI) to work as adjunct / visiting faculty for a specific period of

time in Indian Public funded academic and research institutions.

-The Scheme recognizes the value of collaborative research as a crucial element for information sharing

among researchers for updating and acquiring knowledge and skills, and also to draw different

perspectives to solve a shared problem.

-VAJRA (Visiting Advanced Joint Research) Faculty scheme is launched by the Department of Science and

Technology which enables NRIs and overseas scientific community to participate and contribute to

research and development in India. The Science and Engineering Research Board (SERB), a Statutory

body of the Department will implement the Scheme.

source: http://dst.gov.in/vajra

http://www.vajra-india.in/

www.freeupscmaterials.org

freeupscmaterials.org

Page 10: QUESTION 1. Q.1 ) NISARGRUNA technology is related to Your ... · Exam Title : TS03 CA-JUNE 17 L2 Email : sandeepbiswas221@gmail.com Contact : 8250416813 Civilsdaily Email: hello@civilsdaily.com

Exam Title : TS03 CA-JUNE 17 L2

Email : [email protected]

Contact : 8250416813

Civilsdaily

Email: [email protected] 0

QUESTION 16.

Which of the statements about the City Liveability Index is/are correct?

1. It is released by NITI Aayog .

2. Delhi has topped the index for the consecutive second time.

Select the correct code

a) 1 only

b) 2 only

c) Both 1 and 2

d) Neither 1 nor 2

Correct Answer: D

Your Answer: D

Explanation

-Ministry of Urban Development has launched the City Liveability Index for measuring the quality of life in

116 major cities including smart cities, capital cities and cities with a population of above one million each.

-The Index enables the cities know where they stand in terms of quality of life and the interventions

required to improve the same.

It is a first of its kind Index to be introduced in the country, according to which cities will be assessed on a

comprehensive set of 79 parameters to capture the extent and quality of infrastructure including availability

of roads, education and health care, mobility, employment opportunities, emergency response, grievance

redressal, pollution, availability of open and green spaces, cultural and entertainment opportunities etc.

-Andhra Pradesh has topped the index followed by Odisha, Chattisgarh and Jharkhand.

Source: http://pib.nic.in/newsite/PrintRelease.aspx?relid=166859

www.freeupscmaterials.org

freeupscmaterials.org

Page 11: QUESTION 1. Q.1 ) NISARGRUNA technology is related to Your ... · Exam Title : TS03 CA-JUNE 17 L2 Email : sandeepbiswas221@gmail.com Contact : 8250416813 Civilsdaily Email: hello@civilsdaily.com

Exam Title : TS03 CA-JUNE 17 L2

Email : [email protected]

Contact : 8250416813

Civilsdaily

Email: [email protected] 0

QUESTION 17.

Which of the following countries has become the latest country to join NATO?

a) Estonia

b) Latvia

c) Lithunia

d) Montenegro

Correct Answer: D

Your Answer: D

Explanation

Montenegro formally became the 29th member of the Western military alliance NAT) at a ceremony in

Washington. Montenegros membership will support greater integration, democratic reform, trade, security,

and stability with all of its neighbours.

NATO, also called the North Atlantic Alliance, is an intergovernmental military alliance between several

North American and European states which constitutes a system of collective defence whereby its member

states agree to mutual defense in response to an attack by any external party.

QUESTION 18.

Food regulator FSSAI has come out with a draft regulation for organic food products. Consider the

following statements about FSSAI

1. The Food Safety and Standards Authority of India (FSSAI) has been established under Food Safety and

Standards Act, 2006 for laying down science based standards for articles of food.

2. Ministry of Science and Technology is the Administrative Ministry for the implementation of FSSAI.

Which of the statements given above is/are correct?

a) 1 only

b) 2 only

c) Both 1 and 2

d) Neither 1 nor 2

Correct Answer: A

Your Answer: A

Explanation

The Food Safety and Standards Authority of India (FSSAI) has been established under Food Safety and

Standards Act, 2006 which consolidates various acts & orders that have hitherto handled food related

www.freeupscmaterials.org

freeupscmaterials.org

Page 12: QUESTION 1. Q.1 ) NISARGRUNA technology is related to Your ... · Exam Title : TS03 CA-JUNE 17 L2 Email : sandeepbiswas221@gmail.com Contact : 8250416813 Civilsdaily Email: hello@civilsdaily.com

Exam Title : TS03 CA-JUNE 17 L2

Email : [email protected]

Contact : 8250416813

Civilsdaily

Email: [email protected] 0

issues in various Ministries and Departments.

It was created for laying down science based standards for articles of food and to regulate their

manufacture, storage, distribution, sale and import to ensure availability of safe and wholesome food for

human consumption.

Ministry of Health & Family Welfare, Government of India is the Administrative Ministry for the

implementation of FSSAI.

http://www.thehindubusinessline.com/news/fssai-issues-draft-regulations-for-organic-food-products/article9737535.ece

QUESTION 19.

Government has recently unveiled Open Acreage Licensing Policy (OALP). Which of the following

statements are correct about OALP?

1. It gives exploration companies the option to select the exploration blocks on their own, without having to

wait for the formal bid round from the Government.

2. It is expected to reduce drastically the area of sedimentary basins available for oil exploration and

production activities.

Select the correct code

a) 1 only

b) 2 only

c) Both 1 and 2

d) Neither 1 nor 2

Correct Answer: A

Your Answer: A

Explanation

-The government has unveiled the Open Acreage Licensing Policy (OALP) and the National Data

Repository that is expected to open up 2.8 million square kilometres of sedimentary basins to exploration

and production activities. Hence, it will increase the area and not reduce it.

-The OALP, a part of the governments Hydrocarbon Exploration and Licensing Policy (HELP), gives

exploration companies the option to select the exploration blocks on their own, without having to wait for

the formal bid round from the Government. The company then submits an application to the government,

www.freeupscmaterials.org

freeupscmaterials.org

Page 13: QUESTION 1. Q.1 ) NISARGRUNA technology is related to Your ... · Exam Title : TS03 CA-JUNE 17 L2 Email : sandeepbiswas221@gmail.com Contact : 8250416813 Civilsdaily Email: hello@civilsdaily.com

Exam Title : TS03 CA-JUNE 17 L2

Email : [email protected]

Contact : 8250416813

Civilsdaily

Email: [email protected] 0

which puts that block up for bid.

Tikdams:

How can any Government launch a policy which will significantly reduce the area of sedimentary

basins avaliable for oil exploration.India imports 70% of oil and Gas.Government is trying its best to

improve oil and Gas production in India.Thus there is no chance that statement 2 is correct.

Source:

http://www.thehindu.com/business/Economy/govt-unveils-new-hydrocarbon-policy/article19165558.ece

QUESTION 20.

The mobile app eVIN has been awarded with GSMA Asia Mobile Award. Consider the following statements

about it

1. eVIN is launched by Ministry of Women and Child Development.

2. It will monitor the temperature of the cold chain through a smartphone application.

3. It aims at providing real-time information on vaccine stocks and flows.

Which of the statements given above are correct?

a) 1 and 2

b) 2 and 3

c) 1 and 3

d) 1, 2 and 3

Correct Answer: B

Your Answer: Unanswered

Explanation

-The Electronic Vaccine Intelligence Network (eVIN) is an online real-time vaccine logistics management

system developed and implemented by the Ministry of Health and the United Nations Development

Programme (UNDP).

-It is an indigenously developed technology system in India that digitizes vaccine stocks and monitors the

temperature of the cold chain through a smartphone application.

www.freeupscmaterials.org

freeupscmaterials.org

Page 14: QUESTION 1. Q.1 ) NISARGRUNA technology is related to Your ... · Exam Title : TS03 CA-JUNE 17 L2 Email : sandeepbiswas221@gmail.com Contact : 8250416813 Civilsdaily Email: hello@civilsdaily.com

Exam Title : TS03 CA-JUNE 17 L2

Email : [email protected]

Contact : 8250416813

Civilsdaily

Email: [email protected] 0

-The innovative eVIN is presently being implemented called across twelve states in India. eVIN aims to

support the Government of Indias Universal Immunization Programme by providing real-time information on

vaccine stocks and flows, and storage temperatures across all cold chain points in these states.

Tikdams: Since it is related to vaccine.No way it can be released by Ministry of Women and Child.

Once you eliminate statement 1.You will automatically get the answer using the Tikdam method.

http://www.business-standard.com/article/pti-stories/health-ministry-s-mobile-app-evin-wins-gsma-asia-mobile-award-117062901117_1.html

QUESTION 21.

ISRO has successfully launched the Cartosat-2E satellite. Which of the statements given below about

CARTOSAT series of satellites are correct?

1. It is a version of geostationary satellite.

2. It will provide useful space-based data for town planners, creators of urban infrastructure, for agriculture

and project monitoring, and for decision makers in Smart City and AMRUT projects.

Select the correct code

a) 1 only

b) 2 only

c) Both 1 and 2

d) Neither 1 nor 2

Correct Answer: B

Your Answer: B

Explanation

The satellite, known as Cartosat-2E, is an Indian remote sensing (IRS) or earth observation satellite and

not geostationary satellites. Geostationary satellites are useful for communication purposes.

It will provide useful space-based data for town planners, creators of urban infrastructure, for agriculture

and project monitoring, and for decision makers in Smart City and AMRUT projects.

Source: https://www.nasaspaceflight.com/2017/06/pslv-rocket-cartosat-2e-30-small-sats/

www.freeupscmaterials.org

freeupscmaterials.org

Page 15: QUESTION 1. Q.1 ) NISARGRUNA technology is related to Your ... · Exam Title : TS03 CA-JUNE 17 L2 Email : sandeepbiswas221@gmail.com Contact : 8250416813 Civilsdaily Email: hello@civilsdaily.com

Exam Title : TS03 CA-JUNE 17 L2

Email : [email protected]

Contact : 8250416813

Civilsdaily

Email: [email protected] 0

QUESTION 22.

JNPT has recently been attacked with Petya ransomware. Which of the statements about Petya

ransomware are incorrect?

1. It does not encrypt files on computers, but attacks a part of the Operating System that is called the

Master File Table (MFT) and locks up the data.

2. It forces the victimized Windows users to pay a digital ransom through US dollars in return of their data.

Select the correct code

a) 1 only

b) 2 only

c) Both 1 and 2

d) Neither 1 nor 2

Correct Answer: B

Your Answer: Unanswered

Explanation

-Petya ransomware Petya is a ransomware, similar to the Wannacry ransomware. However, it is a variation

of the WannaCry cyber-attack. It does not encrypt files on computers, but attacks a part of the Operating

System that is called the Master File Table (MFT) and locks up the data.

-It forces the victimized Windows users to pay a digital ransom through Bitcoin in return of their data.

source: https://www.stellarinfo.com/blog/recover-data-petya-ransomware-attack/

http://indianexpress.com/article/technology/tech-news-technology/petya-ransomware-maersk-controlled-terminals-at-jnpt-pipavav-port-hit-4726969/

QUESTION 23.

Consider the following statements about Web App for Legal Services to Prisoners launched by NALSA

www.freeupscmaterials.org

freeupscmaterials.org

Page 16: QUESTION 1. Q.1 ) NISARGRUNA technology is related to Your ... · Exam Title : TS03 CA-JUNE 17 L2 Email : sandeepbiswas221@gmail.com Contact : 8250416813 Civilsdaily Email: hello@civilsdaily.com

Exam Title : TS03 CA-JUNE 17 L2

Email : [email protected]

Contact : 8250416813

Civilsdaily

Email: [email protected] 0

1. It aims at providing free legal services to prisoners so as to ensure that no inmate goes unrepresented in

court.

2. It will allow State and District Legal Services Authorities to feed the data for individual prison inmate in

the jails within their jurisdiction.

Which of the statements given above is/are correct?

a) 1 only

b) 2 only

c) Both 1 and 2

d) Neither 1 nor 2

Correct Answer: C

Your Answer: C

Explanation

Web App for Legal Services to Prisoners was launched for providing free legal services to prisoners so as

to ensure that no inmate goes unrepresented in court.

It will allow State and District Legal Services Authorities to feed the data for individual prison inmate in the

jails within their jurisdiction.

The web application will help in making the legal services system more transparent and monitor the grant

of legal aid to the prisoners in order to make sure that all prisoners are represented right from their first day

in the court.

NALSA has been constituted under the Legal Services Authorities Act, 1987. It provides services of free

legal aid in civil and criminal matters for the poor and marginalised people who cannot afford the services

of a lawyer in any court or tribunal. It also organises Lok Adalats for amicable settlement of disputes.

http://www.newindianexpress.com/nation/2017/jun/29/nalsa-launches-web-app-to-monitor-legal-service-to-prisoners-1621991.html

QUESTION 24.

Which of the following are the advantages of the recently launched portal MERIT ( Merit Order Despatch of

Electricity for Rejuvenation of Income and Transparency)?

1. Disseminates information pertaining to marginal variable cost and source wise purchase of electricity

transparently

2. Promotes economy and efficiency in operations

3. Optimization of the power procurement costs

www.freeupscmaterials.org

freeupscmaterials.org

Page 17: QUESTION 1. Q.1 ) NISARGRUNA technology is related to Your ... · Exam Title : TS03 CA-JUNE 17 L2 Email : sandeepbiswas221@gmail.com Contact : 8250416813 Civilsdaily Email: hello@civilsdaily.com

Exam Title : TS03 CA-JUNE 17 L2

Email : [email protected]

Contact : 8250416813

Civilsdaily

Email: [email protected] 0

4. Demystifies the utility portfolio and its complexity

Select the correct code

a) 1, 2 and 3

b) 2, 3 and 4

c) 1, 3 and 4

d) 1, 2, 3 and 4

Correct Answer: D

Your Answer: D

Explanation

MERIT (Merit Order Despatch of Electricity for Rejuvenation of Income and Transparency) has been

developed by Ministry of Power in association with POSOCO and Central Electricity Authority.

The Web portal provides information regarding the merit order of Electricity procured by States. It will help

state Discoms in optimising their power procurement more efficiently and will help in lowering the cost of

power to consumers.

The main advantages of the portal: Empowerment of consumers and facilitates participative governance;

Disseminates information pertaining to marginal variable cost and source wise purchase of electricity

transparently; Promotes economy and efficiency in operations; Optimization of the power procurement

costs; Demystifies the utility portfolio and its complexity; Facilitates renewable integration and handling of

the variability and uncertainty of renewables; and Indication of supply side reliability, adequacy, and cost of

power procurement.

QUESTION 25.

Consider the following statements about recently launched Energy Conservation Building Code 2017

1. It has been developed by Ministry of Environment, Forests and Climate Change.

2. The code aims to optimise energy savings with the comfort levels for occupants, and prefers life-cycle

cost effectiveness to achieve energy neutrality in commercial buildings.

Which of the statements given above is/are correct?

a) 1 only

b) 2 only

c) Both 1 and 2

d) Neither 1 nor 2

Correct Answer: B

Your Answer: Unanswered

www.freeupscmaterials.org

freeupscmaterials.org

Page 18: QUESTION 1. Q.1 ) NISARGRUNA technology is related to Your ... · Exam Title : TS03 CA-JUNE 17 L2 Email : sandeepbiswas221@gmail.com Contact : 8250416813 Civilsdaily Email: hello@civilsdaily.com

Exam Title : TS03 CA-JUNE 17 L2

Email : [email protected]

Contact : 8250416813

Civilsdaily

Email: [email protected] 0

Explanation

Energy Conservation Building Code 2017 (ECBC 2017) has been developed by Ministry of Power and

Bureau of Energy Efficiency (BEE).

ECBC 2017 prescribes the energy performance standards for new commercial buildings to be constructed

across India. It also sets parameters for builders, designers and architects to integrate renewable energy

sources in building design with the inclusion of passive design strategies.

The code aims to optimise energy savings with the comfort levels for occupants, and prefers life-cycle cost

effectiveness to achieve energy neutrality in commercial buildings.

http://pib.nic.in/newsite/PrintRelease.aspx?relid=166867

QUESTION 26.

According to a survey released by NTCA, Orang Tiger Reserve in Assam has the highest density of tigers

nationally. Consider the following statements about NTCA

1. It is a statutory body under the Ministry of Environment, Forests and Climate Change constituted under

the Environment (Protection) Act, 1972,

2. No alteration in the boundaries of a tiger reserve can be made except on a recommendation of the

NTCA and the approval of the National Board for Wild Life.

Which of the statements given above is/are correct?

a) 1 only

b) 2 only

c) Both 1 and 2

d) Neither 1 nor 2

Correct Answer: B

Your Answer: C

Explanation

The National Tiger Conservation Authority is a statutory body under the Ministry of Environment, Forests

and Climate Change constituted under enabling provisions of the Wildlife (Protection) Act, 1972, as

amended in 2006, for strengthening tiger conservation.

Tiger reserves in India are administered by field directors as mandated by NTCA.

No alteration in the boundaries of a tiger reserve shall be made except on a recommendation of the NTCA

and the approval of the National Board for Wild Life.

No State Government shall de-notify a tiger reserve, except in public interest with the approval of the NTCA

and the approval of the National Board for Wild Life.

www.freeupscmaterials.org

freeupscmaterials.org

Page 19: QUESTION 1. Q.1 ) NISARGRUNA technology is related to Your ... · Exam Title : TS03 CA-JUNE 17 L2 Email : sandeepbiswas221@gmail.com Contact : 8250416813 Civilsdaily Email: hello@civilsdaily.com

Exam Title : TS03 CA-JUNE 17 L2

Email : [email protected]

Contact : 8250416813

Civilsdaily

Email: [email protected] 0

QUESTION 27.

Consider the following statements about Global Innovation Index 2017

1. It is released by Cornell University, INSEAD and World Intellectual Property Organisation (WIPO).

2. Indias performance has fallen down drastically compared to the last year.

3. India is behind all BRICS countries when compared on this index.

Which of the statements given above is/are correct?

a) 1 and 2

b) 1 only

c) 2 and 3

d) 1,2 and 3

Correct Answer: B

Your Answer: B

Explanation

In the Global Innovation Index 2017, co-authored by Cornell University, INSEAD and World Intellectual

Property Organisation (WIPO), India has moved up 6 places to reach 60th place among 130 countries

emerging as the top-ranked economy in Central and South Asia.

Switzerland leads the rankings for the 7th consecutive year.

Among Indias neighbours, Sri Lanka has managed to bag 90th spot and Nepal stood at 109th place.

Pakistan has been ranked as 113th followed by Bangladesh at 114.

BRICS countries are rated as: Brazil (69), Russia (45), China (22), South Africa (57). India (60).

QUESTION 28.

Asian Infrastructure Investment Bank has approved $150 million equity investment loan to the India

Infrastructure Fund. Consider the following statements about AIIB

1. AIIB is a multilateral development bank set up to complement the efforts of World Bank and Asian

Development Bank (ADB).

2. India is the largest shareholder in the bank.

3. It is headquartered at Shanghai.

www.freeupscmaterials.org

freeupscmaterials.org

Page 20: QUESTION 1. Q.1 ) NISARGRUNA technology is related to Your ... · Exam Title : TS03 CA-JUNE 17 L2 Email : sandeepbiswas221@gmail.com Contact : 8250416813 Civilsdaily Email: hello@civilsdaily.com

Exam Title : TS03 CA-JUNE 17 L2

Email : [email protected]

Contact : 8250416813

Civilsdaily

Email: [email protected] 0

4. It has been established to fund various infrastructure projects including energy, transportation, urban

construction and logistics as well as education and healthcare all across the world.

Which of the statements given above are incorrect?

a) 1 and 2

b) 2 and 3

c) 1, 3 and 4

d) 1, 2, 3 and 4

Correct Answer: D

Your Answer: C

Explanation

AIIB is a multilateral development bank backed by China and is seen as a rival to the World Bank and

Asian Development Bank (ADB).

The bank was established to fund various infrastructure projects including energy, transportation, urban

construction and logistics as well as education and healthcare in Asia-Pacific region.

The AIIB was officially established with 57 founding members. But as of May 2017, the bank has 52

members. It has an authorised capital of USD 100 billion. The bank is headquartered in Beijing.

Voting Share: China is the largest shareholder with 26.06% voting shares. India is the second largest

shareholder with 7.5% voting shares followed by Russia 5.93% and Germany with 4.5%.

http://www.financialexpress.com/economy/aiib-grants-150-mn-loan-for-india-infrastructure-fund/720244/

QUESTION 29.

According to a study conducted by the UN International Fund for Agricultural Development (IFAD), which of

the following countries has been ranked as the top remittance-receiving country?

a) China

b) India

c) Bangladesh

d) Philippines

Correct Answer: B

Your Answer: B

Explanation

www.freeupscmaterials.org

freeupscmaterials.org

Page 21: QUESTION 1. Q.1 ) NISARGRUNA technology is related to Your ... · Exam Title : TS03 CA-JUNE 17 L2 Email : sandeepbiswas221@gmail.com Contact : 8250416813 Civilsdaily Email: hello@civilsdaily.com

Exam Title : TS03 CA-JUNE 17 L2

Email : [email protected]

Contact : 8250416813

Civilsdaily

Email: [email protected] 0

According to the One Family at a Time study conducted by the UN International Fund for Agricultural

Development (IFAD), Indians working across the world had sent home USD 62.7 billion in 2016 making

India the top remittance-receiving country surpassing China.

Top Remittance Receiving Countries: India (USD 62.7 billion), China (USD 61 billion), the Philippines (USD

30 billion) and Pakistan (USD 20 billion).

source:

http://indianexpress.com/article/world/india-top-remittance-receiving-country-in-2016-un-report-4705011/

QUESTION 30.

According to RBI, Indias foreign exchange (Forex) reserves have increased to a lifetime high of $381.167

billion. Which of the following constitute Forex reserves in India?

1. Foreign currency assets (FCAs)

2. Gold

3. Special Drawing Rights (SDRs)

4. RBIs Reserve position with International Monetary Fund (IMF)

Select the correct code

a) 1 and 2

b) 1, 2 and 3

c) 1, 2 and 4

d) 1, 2, 3 and 4

Correct Answer: D

Your Answer: D

Explanation

According to RBI, Indias foreign exchange (Forex) reserves have increased by \(2.404 billion to touch a

lifetime high of \) 381.167 billion in the week that ended on June 2. The increase was due to increase in

foreign currency assets (FCAs). The components of Indias Foreign Exchange Reserves include: Foreign

currency assets (FCAs), Gold, Special Drawing Rights (SDRs) and RBIs Reserve position with

International Monetary Fund (IMF). Out of all the components, FCAs constitute the largest component of

the Forex Reserves.

QUESTION 31.

www.freeupscmaterials.org

freeupscmaterials.org

Page 22: QUESTION 1. Q.1 ) NISARGRUNA technology is related to Your ... · Exam Title : TS03 CA-JUNE 17 L2 Email : sandeepbiswas221@gmail.com Contact : 8250416813 Civilsdaily Email: hello@civilsdaily.com

Exam Title : TS03 CA-JUNE 17 L2

Email : [email protected]

Contact : 8250416813

Civilsdaily

Email: [email protected] 0

The Union Ministry of Law and Justice has cleared the Prevention of Cruelty to Animals Bill 2017 legalising

kambala . With which of the following states is kambala associated?

a) Maharashtra

b) Kerala

c) Tamilnadu

d) Karnataka

Correct Answer: D

Your Answer: D

Explanation

The Union Ministry of Law and Justice has cleared the Prevention of Cruelty to Animals (Karnataka

Amendment) Bill 2017, thus legalising kambala.

Kambala is the traditional slush track buffalo race that is held annually in coastal districts of Karnataka to

entertain rural people of the area.

QUESTION 32.

Consider the following statements regarding Comprehensive Online Modified Modules on Induction

Training (COMMIT)?

1.It is to improve the public service delivery mechanism and provide citizen centric administration

2.It is developed by DoPT in collaboration with United Nations Development Programme (UNDP)

Which of the statements given above is/are correct?

a) 1 only

b) 2 only

c) Both 1 and 2

d) Neither 1 nor 2

Correct Answer: C

Your Answer: Unanswered

Explanation

-Comprehensive Online Modified Modules on Induction Training(COMMIT) is a new training programme for

State Government officials.

-It is to improve the public service delivery mechanism and provide citizen centric administration through

capacity building of officials who interact with the citizens on day-to-day basis.

-Details The COMMIT programme has been developed by DoPT in collaboration with United Nations

www.freeupscmaterials.org

freeupscmaterials.org

Page 23: QUESTION 1. Q.1 ) NISARGRUNA technology is related to Your ... · Exam Title : TS03 CA-JUNE 17 L2 Email : sandeepbiswas221@gmail.com Contact : 8250416813 Civilsdaily Email: hello@civilsdaily.com

Exam Title : TS03 CA-JUNE 17 L2

Email : [email protected]

Contact : 8250416813

Civilsdaily

Email: [email protected] 0

Development Programme (UNDP). It will supplement the existing ITP (Induction Training Program)

launched in 2014-15 for newly recruited state Government officials to develop in them Generic & Domain

specific competencies.

source: http://pib.nic.in/newsite/PrintRelease.aspx?relid=166988

QUESTION 33.

What is TELE-LAW INITIATIVE recently in news?

a) a ) It is an initiative of lawyers to provide legal aid to marginalized communities through

video-conferencing.

b) It is governments initiative to make legal aid easily accessible to the marginalized communities and

citizens living in rural areas through Common Service Centres.

c) It is an initiative of United Nations to provide legal services on international matters to under-developed

countries.

d) It is a part of governments SWAYAM initiative to provide online lectures in LLB.

Correct Answer: B

Your Answer: B

Explanation

It is governments initiative to make legal aid easily accessible to the marginalized communities and citizens

living in rural areas through Common Service Centres .Union Ministry of Law and Justice has partnered

with the Ministry of Electronics and Information Technology, to provide legal aid services through its

Common Service Centres (CSC) at the Panchayat level across India.

source:

http://www.thehindubusinessline.com/economy/policy/us-junks-bilateral-investment-treaty-talks/article9740501.ece

QUESTION 34.

The bilateral investment treaties BITs seeks to protect the interests of foreign investors. India has framed a

new model BIT. Consider the following statements in this regard

www.freeupscmaterials.org

freeupscmaterials.org

Page 24: QUESTION 1. Q.1 ) NISARGRUNA technology is related to Your ... · Exam Title : TS03 CA-JUNE 17 L2 Email : sandeepbiswas221@gmail.com Contact : 8250416813 Civilsdaily Email: hello@civilsdaily.com

Exam Title : TS03 CA-JUNE 17 L2

Email : [email protected]

Contact : 8250416813

Civilsdaily

Email: [email protected] 0

1.The new Indian Model BIT will provide appropriate protection to foreign investors in India and Indian

investors in the foreign country.

2. Under the new model BIT, the Most Favoured Nation status is strengthened

Which of the statements given above is/are correct?

a) 1 only

b) 2 only

c) Both 1 and 2

d) Neither 1 nor 2

Correct Answer: A

Your Answer: A

Explanation

Indias first BIT was signed with UK in 1994.

Indias model draft on BIT.

The new Indian Model BIT text will provide appropriate protection to foreign investors in India and Indian

investors in the foreign country, in the light of relevant international precedents and practices, while

maintaining a balance between the investor's rights and the Government obligations.Some of its provisions

are

1.Deleting the MFN clause.

2. Enterprises based definition of investment Investors who do not set up an enterprise in India to carry

business cannot seek protection under BIT.

3. Compulsorily exhausting the local courts first before approaching international tribunal for dispute

resolution.

4. List of subject exceptions where provisions of BIT would be invalid are health, environment etc.

QUESTION 35.

What are Scorpenes ?

1. The Scorpne -class submarines are a class of diesel-electric attack submarines.

2. It features diesel propulsion and an additional air-independent propulsion (AIP).

3. It is designed and developed by Russian defence PSU.

www.freeupscmaterials.org

freeupscmaterials.org

Page 25: QUESTION 1. Q.1 ) NISARGRUNA technology is related to Your ... · Exam Title : TS03 CA-JUNE 17 L2 Email : sandeepbiswas221@gmail.com Contact : 8250416813 Civilsdaily Email: hello@civilsdaily.com

Exam Title : TS03 CA-JUNE 17 L2

Email : [email protected]

Contact : 8250416813

Civilsdaily

Email: [email protected] 0

Which of the following statements are true?

a) 1 and 2

b) 2 and 3

c) All of the above

d) 1 and 3

Correct Answer: A

Your Answer: A

Explanation

-The Scorpne-class submarines are a class of diesel-electric attack submarines jointly developed by the

French Direction des Constructions Navales (DCN) and the Spanish company Navantia, and now by

DCNS. It features diesel propulsion and an additional air-independent propulsion (AIP). -In 2005, India

chose the Scorpne design; purchasing six submarines for US \(3 billion (US\) 500 million per boat). Under a

technology transfer agreement, the state-owned Mazagon Docks in Mumbai will manufacture the

submarines, and deliver them between 2012 and 2016, however the project is running four years behind

schedule. Why in news? Recently The Indian Navy's newest Scorpene-class submarine, INS Kalvari, has

successfully carried out the test-firing of its underwater torpedo missile source:

http://www.thehindu.com/todays-paper/tp-national/ins-kalvari-to-join-navy-by-july/article18685221.ece

QUESTION 36.

What is Lopinavir ?

a) A TB drug

b) A child-friendly HIV drug

c) A new HIV drug that is 100% effective

d) Drug for treating Diphtheria, Pneumonia and Tuberculosis

Correct Answer: B

Your Answer: B

Explanation

Lopinavir syrup is a child-friendly HIV drug that was in news because it ran out of stock

Source: http://www.civilsdaily.com/child-friendly-hiv-drug-gets-govt-nod/

QUESTION 37.

Consider the following statements:

1. KELT-9b is the coldest planet discovered.

www.freeupscmaterials.org

freeupscmaterials.org

Page 26: QUESTION 1. Q.1 ) NISARGRUNA technology is related to Your ... · Exam Title : TS03 CA-JUNE 17 L2 Email : sandeepbiswas221@gmail.com Contact : 8250416813 Civilsdaily Email: hello@civilsdaily.com

Exam Title : TS03 CA-JUNE 17 L2

Email : [email protected]

Contact : 8250416813

Civilsdaily

Email: [email protected] 0

2. An exoplanet or extrasolar planet is a planet outside our solar system that orbits a star.

3. Since it is tidally locked to its star, the day side of the planet is perpetually bombarded by stellar

radiation.

Which of the following is/are correct?

a) 1 only

b) 2 and 3

c) 1,2 and 3

d) None of the above

Correct Answer: B

Your Answer: B

Explanation

KELT-9b is the hottest planet discovered. An exoplanet or extrasolar planet is a planet outside our solar

system that orbits a star. Since it is tidally locked to its star, the day side of the planet is perpetually

bombarded by stellar radiation.

Source: http://www.civilsdaily.com/hottest-planet-in-universe-discovered/

QUESTION 38.

Consider the following statements:

1. The AIP module is being developed by Indian Space Research Organization.

2. The module enables conventional submarines to stay remain underwater for a longer duration.

Which of the following statements are correct?

a) 1 only

b) 2 only

c) Both 1 and 2

d) Neither 1 nor 2

Correct Answer: B

Your Answer: B

Explanation

The AIP module is being developed by Defence Research and Development Organisation (DRDO). The

module enables conventional submarines to stay remain underwater for a longer duration.

www.freeupscmaterials.org

freeupscmaterials.org

Page 27: QUESTION 1. Q.1 ) NISARGRUNA technology is related to Your ... · Exam Title : TS03 CA-JUNE 17 L2 Email : sandeepbiswas221@gmail.com Contact : 8250416813 Civilsdaily Email: hello@civilsdaily.com

Exam Title : TS03 CA-JUNE 17 L2

Email : [email protected]

Contact : 8250416813

Civilsdaily

Email: [email protected] 0

Source: http://www.civilsdaily.com/scorpene-submarines-to-join-navy-without-aip-modules/

QUESTION 39.

Which of the statements given below about Antarctic Treaty is correct?

a) a ) The Antarctic Treaty applies to the area north of 60 south latitude , including all ice shelves and

islands

b) The Antarctic Treaty applies to the area north of 60 south latitude , but does not include all ice shelves

and islands

c) The Antarctic Treaty applies to the area south of 60 south latitude , including all ice shelves and islands

d) The Antarctic Treaty applies to the area south of 60 south latitude , but does not include ice shelves and

islands

Correct Answer: C

Your Answer: C

Explanation

The Antarctica Treaty applies to the area south of 60 south latitude, including all ice shelves and islands

Source: http://www.civilsdaily.com/a-law-this-time-for-antarctica/

QUESTION 40.

Nikshay , a web-based application is associated with which disease?

a) Tuberculosis

b) Zika virus

c) Malaria

d) Syphilis

Correct Answer: A

Your Answer: A

Explanation

Since 2012, the government requires that patients diagnosed with TB and availing treatment be registered

with Nikshay. Nikshay is a web-based application used by the authorities to track funds, treatment

outcomes and health providers connected to the Revised National Tuberculosis Control Programme.

Source:

http://www.huffingtonpost.in/2017/06/22/the-govt-just-made-aadhaar-mandatory-for-cash-benefits-to-tb-pat_a_22544699/

QUESTION 41.

www.freeupscmaterials.org

freeupscmaterials.org

Page 28: QUESTION 1. Q.1 ) NISARGRUNA technology is related to Your ... · Exam Title : TS03 CA-JUNE 17 L2 Email : sandeepbiswas221@gmail.com Contact : 8250416813 Civilsdaily Email: hello@civilsdaily.com

Exam Title : TS03 CA-JUNE 17 L2

Email : [email protected]

Contact : 8250416813

Civilsdaily

Email: [email protected] 0

Consider the following statements about Tea Board:

1. It was established by the enactment of the Tea Act in 1953 with its headquarters in Kolkata

2. It is a state agency of the Government of Kolkata

3. It is established to promote the cultivation, processing, and domestic trade as well as export of tea from

India

Which of the following are correct?

a) 1 only

b) 1,2 and 3

c) 1 and 3 only

d) 2 only

Correct Answer: C

Your Answer: C

Explanation

It is a state agency of the Government of India. It is established to promote the cultivation, processing, and

domestic trade as well as export of tea from India. It was established by the enactment of the Tea Act in

1953 with its headquarters in Kolkata. The Tea Board India is responsible for the assignment of certification

numbers to exports of certain tea merchants.

Source: http://www.civilsdaily.com/tea-board-wants-100-of-output-sold-via-auctions/

QUESTION 42.

Ransomware is carried out using:

a) A Trojan

b) A Spam

c) Boot virus

d) Encrypted virus

Correct Answer: A

Your Answer: Unanswered

Explanation

Source: http://www.civilsdaily.com/held-to-ransomware/

QUESTION 43.

www.freeupscmaterials.org

freeupscmaterials.org

Page 29: QUESTION 1. Q.1 ) NISARGRUNA technology is related to Your ... · Exam Title : TS03 CA-JUNE 17 L2 Email : sandeepbiswas221@gmail.com Contact : 8250416813 Civilsdaily Email: hello@civilsdaily.com

Exam Title : TS03 CA-JUNE 17 L2

Email : [email protected]

Contact : 8250416813

Civilsdaily

Email: [email protected] 0

The term 'Anti-profiteering' sometimes seen in news, is related to:

a) WTO subsidies

b) Defence procurements

c) Goods and service tax

d) New hydrocarbon policy

Correct Answer: C

Your Answer: C

Explanation

As per Section 171 of the CGST/SGST Act, any reduction in tax rate on any supply of goods or services, or

any benefit of input tax credit, must be passed on to the recipient (for example, customer) by the registered

person (e.g., trader) through a commensurate reduction in prices.

Thus, if a trader is paying, say, Rs 100 less in the new tax rate on a certain item, he has to compulsorily

sell that item for Rs 100 cheaper, so the customer benefits proportionally. Failure to do so would mean the

trader is indulging in profiteering.

QUESTION 44.

Consider the following:

1. Service Tax

2. Value Added Tax (VAT)

3. Entertainment Tax

Which of the above will be subsumed by GST?

a) 1 only

b) 2 and 3

c) 1 and 2

d) All of the above

Correct Answer: D

Your Answer: D

Explanation

All of the above taxes will be subsumed under GST.

QUESTION 45.

www.freeupscmaterials.org

freeupscmaterials.org

Page 30: QUESTION 1. Q.1 ) NISARGRUNA technology is related to Your ... · Exam Title : TS03 CA-JUNE 17 L2 Email : sandeepbiswas221@gmail.com Contact : 8250416813 Civilsdaily Email: hello@civilsdaily.com

Exam Title : TS03 CA-JUNE 17 L2

Email : [email protected]

Contact : 8250416813

Civilsdaily

Email: [email protected] 0

Consider the following statements:

1.Central Board of trustees assists EPFO in administering a compulsory contributory Provident Fund

Scheme

2. EPFO is established in accordance with directive principles of state policy.

a) 1 only

b) 2 only

c) Both 1 and 2

d) Neither 1 nor 2

Which of the above is/are correct?

Correct Answer: B

Your Answer: C

Explanation

EPFO assists Central board of trustees and not vice versa. The Employees' Provident Fund Organisation

(abbreviated to EPFO), is an Organization tasked to assist the Central Board of Trustees, a statutory body

formed by the Employees' Provident Fund and Miscellaneous Provisions Act, 1952.

The Constitution of India under "Directive Principles of State Policy" provides that the State shall within the

limits of its economic capacity make effective provision for securing the right to work, to education and to

public assistance in cases of unemployment, old-age, sickness & disablement and undeserved want. The

EPF & MP Act, 1952 was enacted by the Parliament of India as part of a series of legislative interventions

made in this direction.

Source: http://www.civilsdaily.com/unclaimed-pf-to-fund-medical-costs/

QUESTION 46.

Consider the following statements regarding Bureau of Energy Efficiency (BEE).

1. It is a statutory body under Ministry of Environment

2. It develops the Energy Performance Index (EPI)

3. It is one of the primary Green building rating agencies in India

Select the correct answer using the code given below:

a) 1,2 only

b) 2,3 only

c) 1,3 only

d) 1,2 and 3

www.freeupscmaterials.org

freeupscmaterials.org

Page 31: QUESTION 1. Q.1 ) NISARGRUNA technology is related to Your ... · Exam Title : TS03 CA-JUNE 17 L2 Email : sandeepbiswas221@gmail.com Contact : 8250416813 Civilsdaily Email: hello@civilsdaily.com

Exam Title : TS03 CA-JUNE 17 L2

Email : [email protected]

Contact : 8250416813

Civilsdaily

Email: [email protected] 0

Correct Answer: B

Your Answer: Unanswered

Explanation

The Bureau of Energy Efficiency is an agency of the Government of India, under the Ministry of Power

created in March 2002 under the provisions of the nation's 2001 Energy Conservation Act. The agency's

function is to develop programs which will increase the conservation and efficient use of energy in India.

QUESTION 47.

Who designates biodiversity hotspot?

a) International Union for Conservation of Nature(IUCN)

b) United Nations Environment Programme(UNEP)

c) Conservation International

d) World Wide Fund for Nature

Correct Answer: C

Your Answer: C

Explanation

Conservation International is an American nonprofit environmental organization headquartered in Arlington,

Virginia. Its goal is to protect nature as a source of food, fresh water, livelihoods and a stable climate.

Conservational International (American NGO) designates biodiversity hotspot

QUESTION 48.

Consider the following statements regarding Critical Ecosystem Partnership Fund (CEPF)

1. It is a global program that provides funding and technical assistance to the NGOs and other private

sector partners to protect critical ecosystems.

2. CEPF is a joint initiative of the International Union for Conservation of Nature(IUCN) and the United

Nations Environment Programme(UNEP) and International Monetary Fund (IMF)

Select the correct answer using the code given below:

a) 1 only

b) 2 only

c) Both 1 and 2

d) Neither 1 nor 2

Correct Answer: A

Your Answer: Unanswered

Explanation

www.freeupscmaterials.org

freeupscmaterials.org

Page 32: QUESTION 1. Q.1 ) NISARGRUNA technology is related to Your ... · Exam Title : TS03 CA-JUNE 17 L2 Email : sandeepbiswas221@gmail.com Contact : 8250416813 Civilsdaily Email: hello@civilsdaily.com

Exam Title : TS03 CA-JUNE 17 L2

Email : [email protected]

Contact : 8250416813

Civilsdaily

Email: [email protected] 0

-Critical Ecosystem Partnership Fund (CEPF) is a global program that provides funding and technical

assistance to non-governmental organizations and other private sector partners to protect critical

ecosystems. They focus on biodiversity hotspots, the Earth's biologically richest yet most endangered

areas.

-CEPF is a joint initiative of l'Agence Franaise de Dveloppement, Conservation International, the European

Union, the Global Environment Facility, the Government of Japan, the MacArthur Foundation and the World

Bank.

source:

http://www.thehindu.com/todays-paper/tp-national/tp-andhrapradesh/burrowing-frogs-add-to-diversity-of-western-ghats/article19106382.ece

QUESTION 49.

Consider the following statements regarding Nilgiri Thar?

1. It is the state animal of Tamil Nadu.

2. It is endemic to eastern Ghats only

3. Protected (Schedule I) by the Indian Wildlife (Protection) Act of 1972.

4. IUCN status: Endangered

Select the correct answer using the code given below

a) 1,2,3 only

b) 1,3,4only

c) 2,3,4 only

d) All of the above

Correct Answer: B

Your Answer: Unanswered

Explanation

Nilgiri thar is the state animal of Tamil Nadu.

Endemic: Western Ghats from the Nilgiris to Kanyakumari.

IUCN status: Endangered status (because number fewer than 2,500 mature individuals).

www.freeupscmaterials.org

freeupscmaterials.org

Page 33: QUESTION 1. Q.1 ) NISARGRUNA technology is related to Your ... · Exam Title : TS03 CA-JUNE 17 L2 Email : sandeepbiswas221@gmail.com Contact : 8250416813 Civilsdaily Email: hello@civilsdaily.com

Exam Title : TS03 CA-JUNE 17 L2

Email : [email protected]

Contact : 8250416813

Civilsdaily

Email: [email protected] 0

Protected (Schedule I) by the Indian Wildlife (Protection) Act of 1972.

Confined to a narrow belt of higher elevation of Shola Forest.

Biodiversity

source:

http://www.thehindu.com/news/national/kerala/survey-puts-nilgiri-tahr-population-at-1420/article18723761.ece

QUESTION 50.

Multilateral Convention to Implement Tax Treaty Related Measures to Prevent Base Erosion and Profit

Shifting.

1.The Convention is an outcome of the OECD / G20 BEPS Project

2.It is binding upon ratification.

Select the correct answer using the code given below

a) 1 only

b) 2 only

c) Both 1 and 2

d) Neither 1 nor 2

Correct Answer: C

Your Answer: A

Explanation

1.The Multilateral Convention aims at the swift and consistent implementation of the treaty related BEPS

measures.

2. The Convention is an outcome of the OECD / G20 BEPS Project.

3.The convention was conceived as a multilateral instrument which would swiftly modify all covered

bilateral tax treaties to implement BEPS measures.

4.The Convention will have the effect of amending most of the bilateral tax treaties of the signatory

jurisdictions in respect of treaty related measures under the BEPS package for preventing artificial tax

avoidance, prevention of treaty abuse, and improve dispute resolution

5. It will become binding only upon ratification.

source:

www.freeupscmaterials.org

freeupscmaterials.org

Page 34: QUESTION 1. Q.1 ) NISARGRUNA technology is related to Your ... · Exam Title : TS03 CA-JUNE 17 L2 Email : sandeepbiswas221@gmail.com Contact : 8250416813 Civilsdaily Email: hello@civilsdaily.com

Exam Title : TS03 CA-JUNE 17 L2

Email : [email protected]

Contact : 8250416813

Civilsdaily

Email: [email protected] 0

http://www.thehindubusinessline.com/economy/beps-india-signs-historical-multilateral-instrument/article9722568.ece

QUESTION 51.

Which of the following are correctly matched?

Tiger Reserve : State

1. Orang tiger reserve : Arunachal Pradesh

2. Kamlang Tiger Reserve : Assam

3. Mudumallai tiger reserve : Kerala

4. Bandhavgarh tiger reserve: Karnataka

Select the correct answer using the code given below

a) 1 and 4 only

b) 1,2 and 3 only

c) 4 only

d) 1,2,4 only

Correct Answer: C

Your Answer: C

Explanation

Orang tiger reserve : Assam

Kamlang Tiger Reserve: Arunachal Pradesh

Mudumallai tiger reserve : Tamil Nadu

Bandhavgarh tiger reserve: Karnataka

Why in news?

-Orang National Park, one of the smallest national parks in India is poised to emerge as the reserve with

the highest tiger density with 35 tigers per 100 sq km.

www.freeupscmaterials.org

freeupscmaterials.org

Page 35: QUESTION 1. Q.1 ) NISARGRUNA technology is related to Your ... · Exam Title : TS03 CA-JUNE 17 L2 Email : sandeepbiswas221@gmail.com Contact : 8250416813 Civilsdaily Email: hello@civilsdaily.com

Exam Title : TS03 CA-JUNE 17 L2

Email : [email protected]

Contact : 8250416813

Civilsdaily

Email: [email protected] 0

QUESTION 52.

Consider the following statements regarding NAG missile?

1. It is the third generation fire and forget anti-tank missile

2. It can be launched from land based platforms only

3. It is one of the five missile systems under the integrated guided missile development programme

(IGMDP)

4. It is developed by ISRO

Select the correct answer using the code given below:

a) 1,3 only

b) 1,2,3 only

c) 2,3 only

d) 2,3,4 only

Correct Answer: A

Your Answer: A

Explanation

The Nag missile is a third generation fire and forget anti-tank missile and can be launched from land and

airbased platforms. It is equipped with highly advanced Imaging Infrared Radar (IRR) seeker and has

integrated avionics technology in its arsenal. It is one of the five missile systems developed by the Defence

Research and Development Organization (DRDO) under the integrated guided missile development

programme (IGMDP). The four other missiles developed under this programme include Agni, Akash,

Trishul and Prithvi.

Tikdams: Again look for extreme words..Land based platform only.eliminate statement 2 and you will

automatically get the answer

source:

http://www.civilsdaily.com/drdo-successfully-test-fires-third-generation-anti-tank-nag-missile-in-rajasthan/

www.freeupscmaterials.org

freeupscmaterials.org

Page 36: QUESTION 1. Q.1 ) NISARGRUNA technology is related to Your ... · Exam Title : TS03 CA-JUNE 17 L2 Email : sandeepbiswas221@gmail.com Contact : 8250416813 Civilsdaily Email: hello@civilsdaily.com

Exam Title : TS03 CA-JUNE 17 L2

Email : [email protected]

Contact : 8250416813

Civilsdaily

Email: [email protected] 0

QUESTION 53.

Consider the following statements regarding GSLV MK III D1 rocket(GSAT 19)?

1. It is the heaviest rocket to be launched from India till now

2. It can lift payloads of up to 10000 kg to Geosynchronous Transfer Orbit and 4000 kg into the Low Earth

Orbit

3. It is a three stage vehicle with an indigenous cryogenic upper stage engine

4. It has been designed to carry heavier communication satellites into the Geosynchronous Transfer Orbit.

Select the correct code:

a) 1,3,4 only

b) 1,2 and 3

c) 2 and 3

d) 2,3 and 4

Correct Answer: A

Your Answer: A

Explanation

It is the heaviest rocket to be launched from India till now. It can lift payloads of up to 4000 kg to

Geosynchronous Transfer Orbit and 10000 kg into the Low Earth Orbit.

It is a three stage vehicle with an indigenous cryogenic upper stage engine (C25). It has been designed to

carry heavier communication satellites into the Geosynchronous Transfer Orbit.

source: http://www.civilsdaily.com/india-successfully-fires-heaviest-launch-vehicle/

QUESTION 54.

Consider the following tribes and their area of settlement:

Tribe Area

1. Munda Jharkhand

2. Gorkha Darjeeling

www.freeupscmaterials.org

freeupscmaterials.org

Page 37: QUESTION 1. Q.1 ) NISARGRUNA technology is related to Your ... · Exam Title : TS03 CA-JUNE 17 L2 Email : sandeepbiswas221@gmail.com Contact : 8250416813 Civilsdaily Email: hello@civilsdaily.com

Exam Title : TS03 CA-JUNE 17 L2

Email : [email protected]

Contact : 8250416813

Civilsdaily

Email: [email protected] 0

3. Angami Nagaland

Which of the above are correctly matched?

a) 3 only

b) 1 and 2 only

c) 2 and 3 only

d) All of the above

Correct Answer: D

Your Answer: D

Explanation

Munda tribe mainly inhabit in the region of Jharkhand, although they are well spread in the states of West

Bengal, Chhatisgarh, Orissa and Bihar. Munda generally means headman of the village. Hunting is the

main occupation of the Mundas tribe.

Angami tribe belongs to the extreme north eastern part of the country, in the state of Nagaland. The total

population of the Angamis is around 12 million. They are quite popular for their woodcraft and artwork.

Sekrenyi is the main festival celebrated among the Angamis in Nagaland.

Gorkhas are indigenous people living all along the Himalayan belt and the North-East states of India. The

Gorkhas inhabit areas in J&K, Himachal, Uttarakhand, Sikkim, Darjeeling, Assam, and other states in the

North- East.

QUESTION 55.

Consider the following statements regarding Council for Advancement of Peoples Action and Rural

Technology (CAPART):

1. It works as a nodal agency for catalyzing and coordinating the emerging partnership between voluntary

organizations and the Government.

2. CAPART is chaired by the CEO of NITI Aayog.

Which of the statements given above is/are correct?

a) 1 only

b) 2 only

c) Both 1 and 2

d) Neither 1 nor 2

www.freeupscmaterials.org

freeupscmaterials.org

Page 38: QUESTION 1. Q.1 ) NISARGRUNA technology is related to Your ... · Exam Title : TS03 CA-JUNE 17 L2 Email : sandeepbiswas221@gmail.com Contact : 8250416813 Civilsdaily Email: hello@civilsdaily.com

Exam Title : TS03 CA-JUNE 17 L2

Email : [email protected]

Contact : 8250416813

Civilsdaily

Email: [email protected] 0

Correct Answer: A

Your Answer: Unanswered

Explanation

Council for Advancement of People's Action and Rural Technology (CAPART) was formed in 1986, as a

nodal agency for catalysing and coordinating the emerging partnership between voluntary organisations

and the Government for sustainable development of rural areas.

CAPART was formed by amalgamating two agencies the 'Council for Advancement of Rural Technology'

(CART) and People's Action for Development India (PADI). CAPART is an autonomous body registered

under the Societies Registration Act 1860,and is functioning under the aegis of the Ministry of Rural

Development, Government of India. Today, this agency is a major promoter of rural development in India,

assisting over 12,000 voluntary organizations across the country in implementing a wide range of

development initiatives.

CAPART is chaired by the Union Minister for Rural Development.

Why in news: Recently, one SC judgment suggested centre to frame a statutory law regarding regulating

the flow of public money to NGOs. It also directed the government to audit nearly 30 lakh NGOs which

receive public funds but do not explain their expenditure.

Thus, Centre framed new accreditation guidelines for NGOs under which government and CAPART will not

only audit NGOS but also blacklist NGOs found doing misreporting of accounts. Both government and

CAPART would also move to file civil suit for recovery of money siphoned off.

QUESTION 56.

Consider the following statements regarding Union territories:

1.The Governor appoints the CM in States but the President appoints the CM and Ministers for UTs, who

will hold office during the Presidents pleasure.

2.Delhi, Puducherry and Daman & Diu are three Union territories having legislative assemblies.

Which of the above statements are incorrect?

a) 1 only

b) 2 only

c) Both 1 and 2

d) Neither 1 nor 2

www.freeupscmaterials.org

freeupscmaterials.org

Page 39: QUESTION 1. Q.1 ) NISARGRUNA technology is related to Your ... · Exam Title : TS03 CA-JUNE 17 L2 Email : sandeepbiswas221@gmail.com Contact : 8250416813 Civilsdaily Email: hello@civilsdaily.com

Exam Title : TS03 CA-JUNE 17 L2

Email : [email protected]

Contact : 8250416813

Civilsdaily

Email: [email protected] 0

Correct Answer: B

Your Answer: B

Explanation

According to Article 244, the President has powers to make regulations for a UT unless there is a

legislature for that State. Even if there is a legislature, the Administrator can reserve it for the assent of

President, who might reject it, except a money bill.

The Governor appoints the CM in States but the President appoints the CM and Ministers for UTs, who will

hold office during the Presidents pleasure.

Only Puducherry and Delhi have legislative assemblies. Daman & Diu is administered by Central

Government.

Why in news: Puducherry LG Kiran Bedi and CM recently locked horns over powers designated to the two

authorities.

Tikdam: In the presidential election, union territories having legislative assemblies are allowed to

vote. If you remembered that,you could have eliminated statement 2 as only 2 union territories of

Delhi and Puducherry vote for the election of President.

QUESTION 57.

Consider the following statements regarding Aadhar:

1.Personal privacy is not exempted under Right to Information Act and thus biometric data can be collected

by government and can be used by anyone in country.

2.Aadhaar (Targeted Delivery of Financial and Other Subsidies, Benefits and Services) Bill, 2016 provides

government unlimited powers wrt using Aadhar in Social welfare schemes.

Which of the above statements is/are correct?

a) 1 only

b) 2 only

c) Both 1 and 2

d) Neither 1 nor 2

Correct Answer: D

www.freeupscmaterials.org

freeupscmaterials.org

Page 40: QUESTION 1. Q.1 ) NISARGRUNA technology is related to Your ... · Exam Title : TS03 CA-JUNE 17 L2 Email : sandeepbiswas221@gmail.com Contact : 8250416813 Civilsdaily Email: hello@civilsdaily.com

Exam Title : TS03 CA-JUNE 17 L2

Email : [email protected]

Contact : 8250416813

Civilsdaily

Email: [email protected] 0

Your Answer: Unanswered

Explanation

Personal privacy is exempted under Right to Information Act along with individual safety. The core

bio-metric information cannot be shared with any person even with the consent of the Aadhaar card holder.

Even, the general information cannot be unlawfully shared.

National Security is the only ground on which a Competent Authority can share this information. Every

decision of the Competent Authority has to be reviewed by a Committee comprising of the Cabinet

Secretary, the Law Secretary and the Secretary, Information Technology before it is given effect

Aadhaar (Targeted Delivery of Financial and Other Subsidies, Benefits and Services) Bill, 2016 provides

government very limited powers wrt using Aadhar in social welfare schemes.

In 2013, the Supreme Court ruled that Aadhaar could not be made mandatory to receive benefits. No one

should be excluded from social welfare scheme, just because of a requirement of Aadhar.

Why in news: Concerns regarding Aadhar data security due to ransomware attacks across the globe.

http://www.civilsdaily.com/held-to-ransomware/#post-97161

http://www.civilsdaily.com/story/aadhaar-cards-the-identity-revolution/

Tikdam: If 1 was correct, peoples data could have been used by anybody. This would directly

infringe upon their right to life which is one of the fundamental rights of an individual. Also, be

cautious of extreme words. Think about what consequences giving unlimited powers to

government can have?

QUESTION 58.

Consider the following statements:

www.freeupscmaterials.org

freeupscmaterials.org

Page 41: QUESTION 1. Q.1 ) NISARGRUNA technology is related to Your ... · Exam Title : TS03 CA-JUNE 17 L2 Email : sandeepbiswas221@gmail.com Contact : 8250416813 Civilsdaily Email: hello@civilsdaily.com

Exam Title : TS03 CA-JUNE 17 L2

Email : [email protected]

Contact : 8250416813

Civilsdaily

Email: [email protected] 0

1. The Comprehensive Convention on International Terrorism is a proposed treaty which intends to

criminalize all forms of international terrorism and deny terrorists, their financiers and supporters access to

funds, arms, and safe havens.

2. India has proposed this convention in 1996.

Which of the above is/are correct?

a) 1 only

b) 2 only

c) Both 1 and 2

d) Neither 1 nor 2

Correct Answer: C

Your Answer: C

Explanation

The Comprehensive Convention on International Terrorism is a proposed treaty which intends to

criminalize all forms of international terrorism and deny terrorists, their financiers and supporters access to

funds, arms, and safe havens. The negotiations for this treaty are currently under way at the United Nations

General Assembly.

India has proposed this convention in 1996 and has since demanded consistently, especially in the wake of

26/11(Terrorist attack at the Taj Hotel, Mumbai on 26.11.2008).

Why in news: India and Spain stressed that there should be zero tolerance to terrorism

They called on the international community to end selective or partial approaches in combating the menace

of terrorism.

http://www.civilsdaily.com/india-spain-call-for-zero-tolerance-to-terrorism/#post-95638

QUESTION 59.

Consider the following:

1. BrahMos cruise missile programme

2. Sukhoi Su-30MKI programme

www.freeupscmaterials.org

freeupscmaterials.org

Page 42: QUESTION 1. Q.1 ) NISARGRUNA technology is related to Your ... · Exam Title : TS03 CA-JUNE 17 L2 Email : sandeepbiswas221@gmail.com Contact : 8250416813 Civilsdaily Email: hello@civilsdaily.com

Exam Title : TS03 CA-JUNE 17 L2

Email : [email protected]

Contact : 8250416813

Civilsdaily

Email: [email protected] 0

3. Ilyushin /HAL Tactical Transport Aircraft

4. Kamov Ka-226

Which of the above are joint military programmes of India- Russia ?

a) 1 and 2 only

b) 1, 3 and 4

c) 1 only

d) All of the above

Correct Answer: D

Your Answer: Unanswered

Explanation

India and Russia have several major joint military programmes including:

BrahMos cruise missile programme

5th generation fighter jet programme

Sukhoi Su-30MKI programme (230+ to be built by Hindustan Aeronautics)

Ilyushin/HAL Tactical Transport Aircraft

Kamov Ka-226 (200 to be made in India under the Make in India initiative)

Why in news: India and Russia signed agreement to set up 2 more units at nuclear power plants in Tamil

Nadu.

http://www.civilsdaily.com/story/foreign-policy-watch-india-russia/#post-95682

QUESTION 60.

SATH program recently in news is related to

a) Promoting the Self Help Groups in rural areas

b) Effective development and coordination of Central Governments programme

c) Initiate transformation in the education and health sectors.

d) Providing financial and technical assistance to young start-up entrepreneurs

www.freeupscmaterials.org

freeupscmaterials.org

Page 43: QUESTION 1. Q.1 ) NISARGRUNA technology is related to Your ... · Exam Title : TS03 CA-JUNE 17 L2 Email : sandeepbiswas221@gmail.com Contact : 8250416813 Civilsdaily Email: hello@civilsdaily.com

Exam Title : TS03 CA-JUNE 17 L2

Email : [email protected]

Contact : 8250416813

Civilsdaily

Email: [email protected] 0

Correct Answer: C

Your Answer: C

Explanation

NITI Aayog has launched SATH, a program providing Sustainable Action for Transforming Human capital

with the State Governments. The vision of the program is to initiate transformation in the education and

health sectors. The program addresses the need expressed by many states for technical support from NITI

.

QUESTION 61.

Consider the following regarding Foreign Contribution (regulation) Act:

1. It can prohibit acceptance and utilisation of foreign contribution.

2. It applies to individuals or associations but not to companies.

Which of the above is/are correct?

a) 1 only

b) 2 only

c) Both 1 and 2

d) Neither 1 nor 2

Correct Answer: A

Your Answer: A

Explanation

The Foreign Contribution (regulation) Act can prohibit acceptance and utilisation of foreign contribution or

foreign hospitality for any activities detrimental to the national interest.

It can regulate the acceptance and utilisation of foreign contribution or foreign hospitality by certain

individuals or associations or companies.

Why in news: More than 10,000 (NGOs) failing to furnish details of their income and expenditure to the

government have come under the scanner of the Home Ministry. They may end up losing the licence

required to receive any foreign grant.

http://www.civilsdaily.com/story/ngos-vs-goi-the-conflicts-and-scrutinies/#post-96675

Tikdam: This act has been put into place mainly to have an eye on foreign donations received by

the companies, media houses, political parties. Hence, statement 2 is obviously incorrect.

www.freeupscmaterials.org

freeupscmaterials.org

Page 44: QUESTION 1. Q.1 ) NISARGRUNA technology is related to Your ... · Exam Title : TS03 CA-JUNE 17 L2 Email : sandeepbiswas221@gmail.com Contact : 8250416813 Civilsdaily Email: hello@civilsdaily.com

Exam Title : TS03 CA-JUNE 17 L2

Email : [email protected]

Contact : 8250416813

Civilsdaily

Email: [email protected] 0

QUESTION 62.

Consider the following:

1. Shampoos

2. Photographic film

3. Capsules

Which of the following given above contain Gelatin ?

a) 3 only

b) 1 only

c) 2 and 3 only

d) All of the above

Correct Answer: D

Your Answer: Unanswered

Explanation

Gelatin is a protein obtained by boiling skin, tendons, ligaments, and/or bones with water. It is usually

obtained from cows or pigs. Gelatin is used in shampoos, face masks, and other cosmetics; as a thickener

for fruit gelatins and puddings (such as Jell-O); in candies, marshmallows, cakes, ice cream, and yogurts;

on photographic film; and in vitamins as a coating and as capsules, and it is sometimes used to assist in

clearing wines.

Why in news: A notice issued by a health ministry expert committee in the first week of June invited

comments from stakeholders about replacing widely-used animal parts-based gelatin capsules with those

derived from cellulose.

http://www.civilsdaily.com/story/pharma-sector/#post-96679

QUESTION 63.

www.freeupscmaterials.org

freeupscmaterials.org

Page 45: QUESTION 1. Q.1 ) NISARGRUNA technology is related to Your ... · Exam Title : TS03 CA-JUNE 17 L2 Email : sandeepbiswas221@gmail.com Contact : 8250416813 Civilsdaily Email: hello@civilsdaily.com

Exam Title : TS03 CA-JUNE 17 L2

Email : [email protected]

Contact : 8250416813

Civilsdaily

Email: [email protected] 0

Consider the following statements regarding Agriculture Insurance Company of India Limited (AIC):

1. Agricultural loans by banks in India are compulsorily insured by the Agricultural Insurance Company of

India.

2. AIC also transacts other insurance businesses directly or indirectly concerning agriculture and its allied

activities.

Which of the statements given above is/are correct?

a) 1 only

b) 2 only

c) Both 1 and 2

d) Neither 1 nor 2

Correct Answer: C

Your Answer: Unanswered

Explanation

Agricultural loans by banks in India are compulsorily insured by the Agricultural Insurance Company of

India (AIC). Its liabilities are back-stopped by the Centre through budgetary support.

AIC has taken over the implementation of National Agricultural Insurance Scheme (NAIS). In addition, AIC

also transacts other insurance businesses directly or indirectly concerning agriculture and its allied

activities.

Why in news: Farm loan waivers.

http://www.civilsdaily.com/story/financial-inclusion-in-india-and-its-challenges/#post-96811

QUESTION 64.

In case the market price for the commodity falls below the announced minimum price due to bumper

production and glut in the market:

a) government does not procure the farmer's produce anymore

b) only state governments do the procurement

c) government agencies purchase the entire quantity offered by the farmers at the announced minimum

price

d) no mechanism exists for any such condition

www.freeupscmaterials.org

freeupscmaterials.org

Page 46: QUESTION 1. Q.1 ) NISARGRUNA technology is related to Your ... · Exam Title : TS03 CA-JUNE 17 L2 Email : sandeepbiswas221@gmail.com Contact : 8250416813 Civilsdaily Email: hello@civilsdaily.com

Exam Title : TS03 CA-JUNE 17 L2

Email : [email protected]

Contact : 8250416813

Civilsdaily

Email: [email protected] 0

Correct Answer: C

Your Answer: C

Explanation

Minimum Support Price (MSP) is price fixed by Government of India to protect the producer - farmers -

against excessive fall in price during bumper production years. The minimum support prices are a

guarantee price for their produce from the Government.

In case the market price for that commodity falls below the announced minimum price due to bumper

production and glut in the market, government agencies purchase the entire quantity offered by the farmers

at the announced minimum price.

Why in news: Farmer protests in various states due to low prices for produce.

Tikdam: MSP is announced to protect the farmers in case when the market price of crops falls. The

government will have to buy all the produce that comes to it at Minimum Support Price.

QUESTION 65.

Comprehensive Environmental Pollution Index (CEPI) is measured by

a) Global Environment Facility (GEF)

b) Central Pollution Control Board

c) National Ganga River Basin Authority

d) Global Green Growth Institute (GGGI)

Correct Answer: B

Your Answer: Unanswered

Explanation

Comprehensive Environmental Pollution Index (CEPI)

Measured by Central Pollution Control Board

Four Indices for CEPI

1.Scale of Industrial activity Observed Value of Pollution in air

2.Scale of exceedance of Environment quality.

3.Health related statistics.

4.Compliance status of Industry

It is a rational number between 0 and 100

www.freeupscmaterials.org

freeupscmaterials.org

Page 47: QUESTION 1. Q.1 ) NISARGRUNA technology is related to Your ... · Exam Title : TS03 CA-JUNE 17 L2 Email : sandeepbiswas221@gmail.com Contact : 8250416813 Civilsdaily Email: hello@civilsdaily.com

Exam Title : TS03 CA-JUNE 17 L2

Email : [email protected]

Contact : 8250416813

Civilsdaily

Email: [email protected] 0

QUESTION 66.

HELP, NELP and OALP are:

a) IMF subsidiaries in Asia, Africa and Europe

b) Government schemes for Women

c) Goods and service tax forms

d) Names of hydrocarbon policies

Correct Answer: D

Your Answer: D

Explanation

Hydrocarbon Exploration and Licensing Policy (HELP), New Exploration Licensing Policy (NELP) and Open

Acreage Licensing Policy (OALP) are names of different policies of government for exploration &

production of Oil & gas in the country

Why in news: Government proposed changes in policies related to oil exploration.

Tikdam: Some time back, the terms HELP and NELP were in news when in March 2016, Hydrocarbon

Exploration and Licensing Policy (HELP) replaced the extant policy regime for exploration and production

of oil and gas -New Exploration Licensing Policy (NELP).

QUESTION 67.

This pass connects state of Sikkim with China's Tibet Autonomous Region. It also forms a part of an

offshoot of the ancient Silk Road. The vegetation here ranges from sub-tropical forest at its base, to a

temperate region, to a wet and dry alpine climate, and finally to cold tundra desert. The pass being

discussed is:

a) Nathu La

b) Zoji La

c) Shipki La

d) Lhasa

www.freeupscmaterials.org

freeupscmaterials.org

Page 48: QUESTION 1. Q.1 ) NISARGRUNA technology is related to Your ... · Exam Title : TS03 CA-JUNE 17 L2 Email : sandeepbiswas221@gmail.com Contact : 8250416813 Civilsdaily Email: hello@civilsdaily.com

Exam Title : TS03 CA-JUNE 17 L2

Email : [email protected]

Contact : 8250416813

Civilsdaily

Email: [email protected] 0

Correct Answer: A

Your Answer: Unanswered

Explanation

Nathu La is a mountain pass in the Himalayas. It connects the Indian state of Sikkim with China's Tibet

Autonomous Region. The pass, at 4,310 m (14,140 ft) above mean sea level, forms a part of an offshoot of

the ancient Silk Road.

Because of the steep elevation increase around the pass, the vegetation graduates from sub-tropical forest

at its base, to a temperate region, to a wet and dry alpine climate, and finally to cold tundra desert devoid of

vegetation. Around Nathu La and the Tibetan side, the region has little vegetation besides scattered

shrubs.

Why in news: India-China conflict in Doklam

QUESTION 68.

Consider the following regarding Indian Council for Research on International Economic Relations

(ICRIER):

1. ICRIER is a statutory body under Ministry of Finance.

2. ICRIER undertakes research for improving India's interface with the global economy.

Which of the above statements is/are correct?

a) 1 only

b) 2 only

c) Both 1 and 2

d) Neither 1 nor 2

Correct Answer: B

Your Answer: B

Explanation

ICRIER is an autonomous, policy-oriented, not-for-profit, economic policy think tank.

ICRIERs main focus is to enhance the knowledge content of policy making by undertaking analytical

research that is targeted at informing Indias policy makers and also at improving the interface with the

www.freeupscmaterials.org

freeupscmaterials.org

Page 49: QUESTION 1. Q.1 ) NISARGRUNA technology is related to Your ... · Exam Title : TS03 CA-JUNE 17 L2 Email : sandeepbiswas221@gmail.com Contact : 8250416813 Civilsdaily Email: hello@civilsdaily.com

Exam Title : TS03 CA-JUNE 17 L2

Email : [email protected]

Contact : 8250416813

Civilsdaily

Email: [email protected] 0

global economy.

QUESTION 69.

Consider the following:

1. Nathu La pass

2. Jelep La pass

3. Siliguri corridor

Which of the above are located in/near Chumbi valley?

a) 1 only

b) 3 only

c) 1 and 3 only

d) All of the above

Correct Answer: D

Your Answer: C

Explanation

Chumbi Valley is a valley in Tibet at the intersection of India (Sikkim), Bhutan and China (Tibet) in the

Himalayas. Two main passes between India and China open up here: the Nathu La Pass and Jelep La

Pass. Chumbi Valley is a dangerous conduit into the slender Siliguri Corridor and a dangerous choke point.

Why in news: India-China conflict at Chumbi valley.

QUESTION 70.

Business Expectation Index is published by:

a) World Bank

b) IMF

c) Reserve Bank of India

d) Central Statistics Office

Correct Answer: C

Your Answer: Unanswered

Explanation

www.freeupscmaterials.org

freeupscmaterials.org

Page 50: QUESTION 1. Q.1 ) NISARGRUNA technology is related to Your ... · Exam Title : TS03 CA-JUNE 17 L2 Email : sandeepbiswas221@gmail.com Contact : 8250416813 Civilsdaily Email: hello@civilsdaily.com

Exam Title : TS03 CA-JUNE 17 L2

Email : [email protected]

Contact : 8250416813

Civilsdaily

Email: [email protected] 0

The Business Expectation Index (BEI) is released with the quarterly Industrial Outlook Survey by the RBI.

The index is calculated as the weighted average of net responses from industry participants on nine select

parameters. Net response is the percentage difference between respondents who are optimistic and those

reporting pessimism.

Parameters used are overall business situation, production, raw material inventory, finished goods

inventory, order book, employment, capacity utilisation, exports and profit margins. The Index lies between

0 and 200, where a reading above 100 indicates expansion.

Why in news: Recent slowdown in manufacturing and release of related industrial data.

QUESTION 71.

Consider the following statements:

1.Keratitis is the inflammation of the eye which may lead to blindness.

2.Keratitis can be caused only by bacteria.

Which of the statements given above is/are correct?

a) 1 only

b) 2 only

c) Both 1 and 2

d) Neither 1 nor 2

Correct Answer: A

Your Answer: A

Explanation

Keratitis is the inflammation of the eye. It starts with redness and itching and might eventually lead to

blindness.

Keratitis can be caused by both bacteria and fungi. Fungi attach themselves to the cornea and release

enzymes that break down the corneal proteins for their nutritional requirements.

Tikdam: Inflammatory diseases could be caused by either bacteria or fungi. Also, read statements

www.freeupscmaterials.org

freeupscmaterials.org

Page 51: QUESTION 1. Q.1 ) NISARGRUNA technology is related to Your ... · Exam Title : TS03 CA-JUNE 17 L2 Email : sandeepbiswas221@gmail.com Contact : 8250416813 Civilsdaily Email: hello@civilsdaily.com

Exam Title : TS03 CA-JUNE 17 L2

Email : [email protected]

Contact : 8250416813

Civilsdaily

Email: [email protected] 0

with words like only, all, except. Extreme statements are generally wrong.

QUESTION 72.

Consider the following statements:

1.India is a founder-member of BIMSTEC.

2.BIMSTEC has no written charter but has a permanent secretariat.

Which of the above statements is/are correct?

a) 1 only

b) 2 only

c) Both 1 and 2

d) Neither 1 nor 2

Correct Answer: C

Your Answer: C

Explanation

India is a founder-member of BIMSTEC. BIMSTEC or Bay of Bengal Initiative for Multisectoral Technical

and Economic Cooperation was formed two decades ago, in June 06, 1997. The BIMSTEC comprises

India, Bangladesh, Bhutan, Myanmar, Nepal, Sri Lanka, and Thailand.

The permanent secretariat of BIMSTEC was established in Dhaka in 2014. BIMSTEC has no written

charter.

Why in news- The Bay of Bengal Initiative for Multi-Sectoral Technical and Economic Cooperation

(BIMSTEC) celebrated its 20th anniversary on June 06, 2017.

QUESTION 73.

Consider the following statements:

1. Siliguri corridor links the north-east with the rest of India and it is the only access point to the northeast.

2. The friendship treaty of 2007 commits India to protecting Bhutans interest and the close coordination

between the two militaries.

www.freeupscmaterials.org

freeupscmaterials.org

Page 52: QUESTION 1. Q.1 ) NISARGRUNA technology is related to Your ... · Exam Title : TS03 CA-JUNE 17 L2 Email : sandeepbiswas221@gmail.com Contact : 8250416813 Civilsdaily Email: hello@civilsdaily.com

Exam Title : TS03 CA-JUNE 17 L2

Email : [email protected]

Contact : 8250416813

Civilsdaily

Email: [email protected] 0

Which of the above statements is/are correct?

a) 1 only

b) 2 only

c) Both 1 and 2

d) Neither 1 nor 2

Correct Answer: C

Your Answer: C

Explanation

-India and Bhutan have shared since 1947, the friendship treaty of 2007 that commits India to protecting

Bhutans interests, and the close coordination between the two militaries.

-The narrow Siliguri corridor links the north-east with the rest of India and it is the only access point to the

northeast. The Corridor is about 500 km from the Chumbi Valley.

Why in news- India-China face off in Doklam

http://www.civilsdaily.com/tension-at-trijunction-india-china-standoff-over-bhutan-clash/

QUESTION 74.

Information related to which of the following is exempted under RTI?

1. National economic interests

2. Relations with foreign states

3. Law enforcement

4. Individual safety

Select the correct answer using the code given below:

a) 1, 3, 4 only

b) 1, 2, 3 only

c) 2, 3 only

d) 1,2,3 and 4

www.freeupscmaterials.org

freeupscmaterials.org

Page 53: QUESTION 1. Q.1 ) NISARGRUNA technology is related to Your ... · Exam Title : TS03 CA-JUNE 17 L2 Email : sandeepbiswas221@gmail.com Contact : 8250416813 Civilsdaily Email: hello@civilsdaily.com

Exam Title : TS03 CA-JUNE 17 L2

Email : [email protected]

Contact : 8250416813

Civilsdaily

Email: [email protected] 0

Correct Answer: D

Your Answer: Unanswered

Explanation

Information related to following is exempted under RTI

1. National security or sovereignty

2. National economic interests

3. Relations with foreign states

4. Law enforcement and the judicial process

5. Cabinet and other decision making documents

6. Trade secrets & commercial confidentiality

7. Individual safety

8. Personal privacy

Why in news:

CIC in its latest order has urged CoA (Committee of administrators) running BCCI to bring BCCI under RTI.

http://www.civilsdaily.com/op-ed-snap-the-price-of-defiance/

QUESTION 75.

Consider the following statements regarding UN Secretary-Generals appointment

1. The UN Secretary-General is appointed by the General Assembly.

2. The Secretary-General's selection is subject to the veto of any of the five permanent members of the

Security Council.

3. No candidate has ever been rejected by the General Assembly

Which of the statements given above is./are correct?

a) 2 only

b) 1 and 3 only

c) 1,2 and 3

d) 1 only

www.freeupscmaterials.org

freeupscmaterials.org

Page 54: QUESTION 1. Q.1 ) NISARGRUNA technology is related to Your ... · Exam Title : TS03 CA-JUNE 17 L2 Email : sandeepbiswas221@gmail.com Contact : 8250416813 Civilsdaily Email: hello@civilsdaily.com

Exam Title : TS03 CA-JUNE 17 L2

Email : [email protected]

Contact : 8250416813

Civilsdaily

Email: [email protected] 0

Correct Answer: C

Your Answer: Unanswered

Explanation

-According to Article 97 of the United Nations Charter, the Security Council first sends a recommendation

to the General Assembly, which will then appoint the Secretary-General.

-The Secretary-General's selection is subject to the veto of any of the five permanent members of the

Security Council. Since 1981, Secretaries-General have been chosen behind closed doors by the Security

Council and then had their names submitted to General Assembly for ratification.

- No candidate has ever been rejected by the General Assembly.

Why in news:

PM made official visit to US and US support for Indias permanent membership of the UN Security Council,

membership of the Nuclear Suppliers Group, the Wassenaar Arrangement, and the Australia Group has

been reiterated

http://www.civilsdaily.com/story/foreign-policy-watch-india-us/

QUESTION 76.

Consider the following statements regarding Vermin animals

1. Any animal which poses a threat to human and their livelihood can be declared Vermin under Schedule

V of Wildlife Protection act 1972.

2. Wildlife Protection Act 1972, empowers every States Chief Wildlife Warden for culling.

Which of the statements given above is/are correct?

a) 1 only

b) 2 only

c) Both 1 and 2

d) Neither 1 nor 2

Correct Answer: C

Your Answer: Unanswered

Explanation

www.freeupscmaterials.org

freeupscmaterials.org

Page 55: QUESTION 1. Q.1 ) NISARGRUNA technology is related to Your ... · Exam Title : TS03 CA-JUNE 17 L2 Email : sandeepbiswas221@gmail.com Contact : 8250416813 Civilsdaily Email: hello@civilsdaily.com

Exam Title : TS03 CA-JUNE 17 L2

Email : [email protected]

Contact : 8250416813

Civilsdaily

Email: [email protected] 0

. Any animal which poses a threat to human and their livelihood especially farming, can be declared Vermin

under Schedule V of Wildlife Protection act 1972.

States can send a list of wild animals to the Centre requesting it to declare them vermin for selective

slaughter.

Wildlife Protection Act 1972, empower every States Chief Wildlife Warden for culling.

Wild boars, nilgai and rhesus monkeys are protected under Schedule II and III, but can be hunted under

specific conditions.

Why in News?

Tamil Nadu government has planned to allow Forest Department personnel to cull the wild boar for a

limited period of time.

http://www.civilsdaily.com/culling-vermins-schedules-wildlife-protection-act-1972/

QUESTION 77.

Consider the following statements regarding Global Green Growth Institution (GGGI)

1. It was established in 2012, at the Rio+20 United Nations Conference on Sustainable Development.

2. India is a founding member and associated with their research work.

Which of the statements given above is/are correct?

a) 1 only

b) 2 only

c) Both 1 and 2

d) Neither 1 nor 2

Correct Answer: A

Your Answer: Unanswered

Explanation

Global Green Growth Institution (GGGI) is a Seoul (South Korea) based inter-governmental organization.

India is not founding member but associated with research work. It established in 2012, at the Rio+20

United Nations Conference on Sustainable Development. The work with UNFCCC provides a framework

for cooperation.

Why in News?

www.freeupscmaterials.org

freeupscmaterials.org

Page 56: QUESTION 1. Q.1 ) NISARGRUNA technology is related to Your ... · Exam Title : TS03 CA-JUNE 17 L2 Email : sandeepbiswas221@gmail.com Contact : 8250416813 Civilsdaily Email: hello@civilsdaily.com

Exam Title : TS03 CA-JUNE 17 L2

Email : [email protected]

Contact : 8250416813

Civilsdaily

Email: [email protected] 0

In June 2017, OECD released a report titled Green Growth Indicators 2017 highlighting the slow progress

in achieving the Green Growth. An Initiative on Green Growth and Development in India is a collaborative

project Global Green Growth Institute (GGGI) and The Energy and Resources Institute (TERI).

QUESTION 78.

Consider the following statements regarding The India Meteorological Department (IMD)?

1. It is under the Ministry of Environment, Forest and Climate Change

2. It is the principal agency responsible for meteorological observations, weather forecasting and

seismology.

3. IMD is also one of the six Regional Specialised Meteorological Centres of the World Meteorological

Organization.

Which of the statements given above is.are correct?

a) 1 only

b) 1,2 only

c) 2 and 3 only

d) 1,2 and 3

Correct Answer: C

Your Answer: C

Explanation

The India Meteorological Department (IMD)

1. IMD established in 1875 and headquartered in New Delhi, is under the Ministry of Earth Sciences.

2. It is the principal agency responsible for meteorological observations, weather forecasting and

seismology. IMD is also one of the six Regional Specialised Meteorological Centres of the World

Meteorological Organization.

Why in news?

The India Meteorological Department (IMD) is working on a forecasting system to give 15-day warnings on

the likelihood of a malaria or chikungunya outbreak, over different regions.

QUESTION 79.

Consider the following statements about Preventive Detention:

1. The power of Preventive Detention is statutory in nature.

www.freeupscmaterials.org

freeupscmaterials.org

Page 57: QUESTION 1. Q.1 ) NISARGRUNA technology is related to Your ... · Exam Title : TS03 CA-JUNE 17 L2 Email : sandeepbiswas221@gmail.com Contact : 8250416813 Civilsdaily Email: hello@civilsdaily.com

Exam Title : TS03 CA-JUNE 17 L2

Email : [email protected]

Contact : 8250416813

Civilsdaily

Email: [email protected] 0

2. Preventive detention can even be resorted to when sufficient remedies are available under the general

laws of the land

3. Recently, the Supreme Court said that preventive detention of a person by a State after branding him a

goonda is legal

Which of the statements given above is/are incorrect?

a) 1 and 2 only

b) 2 and 3 only

c) 1 only

d) 3 only

Correct Answer: B

Your Answer: B

Explanation

The Article 22 (3) of the Indian constitution provides that if a person is arrested or detained under a law

providing for preventive detention, then the protection against arrest and detention under Article 22 (1) and

22 (2) shall not be available. Preventive detention on the other hand is action taken beforehand to prevent

possible commitment of crime. Preventive detention thus is action taken on grounds of suspicion that some

wrong actions may be done by the person concerned. Preventive detention of a person by a State after

branding him a goonda merely because the normal legal process is ineffective and time-consuming in

curbing the evil he spreads is illegal. The power of Preventive Detention is statutory in nature its exercise

has to be within the limitations of the statute, and must be exercised for the purpose the power is conferred.

Preventive detention cannot be resorted to when sufficient remedies are available under the general laws

of the land for any omission or commission under such laws.

http://www.civilsdaily.com/preventive-detention- no-quick- fix/

QUESTION 80.

Odisha is the first State in India to have developed an automatic public address system that can be

activated along its entire coast. Who is financing in this development?

a) IMF

b) World Bank

c) Asian Development Bank

d) State Bank of India

Correct Answer: B

Your Answer: B

Explanation

www.freeupscmaterials.org

freeupscmaterials.org

Page 58: QUESTION 1. Q.1 ) NISARGRUNA technology is related to Your ... · Exam Title : TS03 CA-JUNE 17 L2 Email : sandeepbiswas221@gmail.com Contact : 8250416813 Civilsdaily Email: hello@civilsdaily.com

Exam Title : TS03 CA-JUNE 17 L2

Email : [email protected]

Contact : 8250416813

Civilsdaily

Email: [email protected] 0

Explanation:

Odisha is the first State in India to have developed an automatic public address system that can be

activated along its entire coast. Neighbouring Andhra Pradesh is also building up such capability to face

natural calamities.

http://www.civilsdaily.com/odisha-to-have-first-automatic-coastal-warning-for-disasters-in-india/#post-95717

QUESTION 81.

The first Tri- Services exercise named Indira- 2017 is between:

a) India and Malaysia

b) India and Vietnam

c) India and Russia

d) India and Mauritius

Correct Answer: C

Your Answer: C

Explanation

The two countries also decided to hold the first tri-Services exercises, named Indra-2017. The two nations

decided to upgrade and intensify bilateral defence cooperation through joint manufacture, co-production

and co-development of key military hardware and equipment.

http://www.civilsdaily.com/india-russia-ink-nuclear-plant-pact/

QUESTION 82.

Consider the following statements:

1. GSAT-19 is a communication satellite.

2. The satellite weighs 3,136 kg.

3. It was launched from French Guyana

Which of the statements given above are correct?

a) 1 and 3

b) 1,2 and 3

c) 1 and 2

d) 2 and 3

www.freeupscmaterials.org

freeupscmaterials.org

Page 59: QUESTION 1. Q.1 ) NISARGRUNA technology is related to Your ... · Exam Title : TS03 CA-JUNE 17 L2 Email : sandeepbiswas221@gmail.com Contact : 8250416813 Civilsdaily Email: hello@civilsdaily.com

Exam Title : TS03 CA-JUNE 17 L2

Email : [email protected]

Contact : 8250416813

Civilsdaily

Email: [email protected] 0

Correct Answer: C

Your Answer: C

Explanation

GSAT-19 is a communication satellite. It is expected to enhance Indias communication infrastructure and

was placed into a Geosynchronous Transfer Orbit (GTO). The satellite weighs 3,136 kg. ISRO has used

indigenous cryogenic engines on earlier GSLV flights which were modelled mainly on Russian design. On

this GSLV,

Tikdam: The second statement is very factual since it is almost impossible for aspirant to

remember exact weight of an satellite.However using Tikdam you can solve this question without

knowing the statement 2. Third statement is obviously wrong since it was frequently in news that

G-SAT-19 was the heaviest ever rocket and satellite to be launched from the country". Once we

eliminate statement 3 we will automatically get the answer

Read more at:

http://economictimes.indiatimes.com/articleshow/58993288.cms?utm_source=contentofinterest&utm_medium=text&utm_campaign=cppst

http://www.civilsdaily.com/india-successfully-fires-heaviest-launch-vehicle/

QUESTION 83.

Consider the following statements:

1. Neutrinos help us detect early geological defects deep within the earth that can provide early earthquake

warnings

2. They can help in detecting mineral and oil deposits deep in the earth.

3. INO project primarily aims to study atmospheric neutrinos in Mumbai.

Which of the statements given above are correct?

a) 1 and 2 only

b) 1 and 3 only

c) 1,2 and 3

d) 2 and 3 only

www.freeupscmaterials.org

freeupscmaterials.org

Page 60: QUESTION 1. Q.1 ) NISARGRUNA technology is related to Your ... · Exam Title : TS03 CA-JUNE 17 L2 Email : sandeepbiswas221@gmail.com Contact : 8250416813 Civilsdaily Email: hello@civilsdaily.com

Exam Title : TS03 CA-JUNE 17 L2

Email : [email protected]

Contact : 8250416813

Civilsdaily

Email: [email protected] 0

Correct Answer: A

Your Answer: Unanswered

Explanation

Neutrinos help us detect early geological defects deep within the earth that can provide early earthquake

warnings. They can help in detecting mineral and oil deposits deep in the earth. The proposed INO project

primarily aims to study atmospheric neutrinos in a 1,300-m deep cavern in the Bodi West Hills in Theni

district, Tamil Nadu.

http://www.civilsdaily.com/op-ed-snap-the-neutrino-opportunity/

QUESTION 84.

Consider the following statements

1. Masala bonds are bonds issued outside India in foreign currency

2. The term was used by International Finance Corporation

3. In dollar bonds, the borrower takes the currency risk but masala bond makes the investors bear the risk

4. The first Masala bond was issued by the International Monetary Fund backed International Finance

Corporation in November 2014

5. First Indian Company was HDFC that raised money from Masala bonds

6. NTPC issued first corporate green masala bonds

Which of the statements given above are correct?

a) 1,2,3 and 4

b) 2,3 and 6

c) 2,3,5 and 6

d) 2,4 and 5

Correct Answer: C

Your Answer: D

Explanation

Masala bonds are bonds issued outside India but denominated in Indian Rupees, rather than the local

currency. The term was used by IFC to evoke the culture and cuisine of India. In dollar bonds, the borrower

takes the currency risk but masala bond makes the investors bear the risk. The first Masala bond was

issued by the World Bank backed International Finance Corporation in November 2014. First Indian

Company: In July 2016 HDFC raised 3,000 crore rupees from Masala bonds and thereby became the first

www.freeupscmaterials.org

freeupscmaterials.org

Page 61: QUESTION 1. Q.1 ) NISARGRUNA technology is related to Your ... · Exam Title : TS03 CA-JUNE 17 L2 Email : sandeepbiswas221@gmail.com Contact : 8250416813 Civilsdaily Email: hello@civilsdaily.com

Exam Title : TS03 CA-JUNE 17 L2

Email : [email protected]

Contact : 8250416813

Civilsdaily

Email: [email protected] 0

Indian company to issue masala bonds. First Major Corporate: In the month of August 2016 public sector

unit NTPC issued first corporate green masala bonds worth 2,000 crore rupees

http://www.civilsdaily.com/firms-can-issue-masala-bonds-only-post-rbi-nod/

QUESTION 85.

Consider the following statements about uses of Laser Interferometer Gravitational Wave Observatory

(LIGO):

1. LIGO will help in gravitational wave astronomy

2. Detection of new heavenly bodies

3. Gaining a better understanding of that most elusive(difficult to understand) of theories Einsteins general

theory of relativity, and the fundamental force of gravitation

Which of the following are correct?

a) 1 and 2 only

b) 2 and 3 only

c) 1,2 and 3

d) None of the above

Correct Answer: C

Your Answer: C

Explanation

LIGO will help in gravitational wave astronomy. It will help in detection of new heavenly bodies. Gaining a

better understanding of that most elusive (difficult to understand) of theories Einsteins general theory of

relativity, and the fundamental force of gravitation.

http://www.civilsdaily.com/op-ed-snap-detecting-possibilities/

QUESTION 86.

Consider the following statements about Zero defect zero effect scheme:

1. The scheme is for MSME (Micro, Small and Medium Enterprises) Sector to increase the quality of

products matching global standards

2. The main objective of ZED scheme is to reduce the bad effect of products on environment

3. ZED Scheme aims to rate and handhold all MSMEs to deliver top quality products using clean

technology.

Which of the statements given above is/are incorrect?

www.freeupscmaterials.org

freeupscmaterials.org

Page 62: QUESTION 1. Q.1 ) NISARGRUNA technology is related to Your ... · Exam Title : TS03 CA-JUNE 17 L2 Email : sandeepbiswas221@gmail.com Contact : 8250416813 Civilsdaily Email: hello@civilsdaily.com

Exam Title : TS03 CA-JUNE 17 L2

Email : [email protected]

Contact : 8250416813

Civilsdaily

Email: [email protected] 0

a) 1 and 2

b) 2 and 3

c) 1,2 and 3

d) None of the above

Correct Answer: D

Your Answer: B

Explanation

Zero defect zero effect scheme is for MSME (Micro, Small and Medium Enterprises) Sector to increase the

quality of products matching global standards. The main objective of ZED scheme is to reduce the bad

effect of products on environment. ZED Scheme aims to rate and handhold all MSMEs to deliver top quality

products using clean technology.

http://www.civilsdaily.com/success-the-zed-way/#post-96574

QUESTION 87.

Global Peace Index (GPI) is published by Consider the following about Global Peace Index 2017:

1. It uses 23 qualitative and quantitative indicators to measure the state of peace using three thematic

domains: the level of Societal Safety and Security; the extent of Ongoing Domestic and International

Conflict; and the degree of Militarisation.

2. It is released by Institute for Economics and Peace (IEP), an independent, nonprofit think tank

headquartered in Sydney..

3. Israel is ranked as worlds most peaceful country

Which of the above statement(s) is/are correct?

a) 1, 2 and 3

b) 2 and 3

c) 1 and 2

d) 1 and 3

Correct Answer: C

Your Answer: C

Explanation

Explanation:

Iceland is voted as the most peaceful country.

www.freeupscmaterials.org

freeupscmaterials.org

Page 63: QUESTION 1. Q.1 ) NISARGRUNA technology is related to Your ... · Exam Title : TS03 CA-JUNE 17 L2 Email : sandeepbiswas221@gmail.com Contact : 8250416813 Civilsdaily Email: hello@civilsdaily.com

Exam Title : TS03 CA-JUNE 17 L2

Email : [email protected]

Contact : 8250416813

Civilsdaily

Email: [email protected] 0

Tikdam-This is an uncommon index. Most likely you won,t have heard about it.However you can

still very easily solve this question using Tikdam. Israel with all its dispute with Palestine and the

arab world faces great security threat from both Hamas and Hexbollah.Thus there is no chance it

could be judged as most peaceful country.Once you eliminate statement 3 you automatically get

the answer without knowing the first 2 statements.

Source: The Hindu (dated June 9th)

QUESTION 88.

Which state recently launched My Plant App ,Which will record the number of trees planted, their species

and details of plantation with Forest Department.

a) Gujarat

b) Maharastra

c) Kerela

d) Karnataka

Correct Answer: B

Your Answer: Unanswered

Explanation

A mobile app named My Plant, which would help in recording data about tree plantations in the state further

would assist them in achieving their mission of planting 13 crore trees in 2018

Using this app, people and organisations, social and voluntary organisations, industry and trade

representatives can feed the data about the saplings they have planted with the forest department, which

would help it in creating a database of trees.

This app makes it possible to upload information of all the tasks given to the government departments in

the tree plantation programme, the name of the tree species being planted and the status.

Source: Hindustan Times (dated June 23rd)

QUESTION 89.

www.freeupscmaterials.org

freeupscmaterials.org

Page 64: QUESTION 1. Q.1 ) NISARGRUNA technology is related to Your ... · Exam Title : TS03 CA-JUNE 17 L2 Email : sandeepbiswas221@gmail.com Contact : 8250416813 Civilsdaily Email: hello@civilsdaily.com

Exam Title : TS03 CA-JUNE 17 L2

Email : [email protected]

Contact : 8250416813

Civilsdaily

Email: [email protected] 0

Recently India and Netherlands have signed 3 agreements including a memorandum of understanding

(MoU) on water cooperation. Consider the following about Indo-Netherland:

1. Netherlands is the 5th largest investment partner globally.

2. The year 2017 marks the 70 year of Indo-Dutch diplomatic relations.

3. Netherland is the largest source of FDI for India.

Which of the above statement(s) is/are correct?

a) 1 and 2 only

b) 2 and 3 only

c) 1 and 3 only

d) All the above

Correct Answer: A

Your Answer: Unanswered

Explanation

The relationship between India and Netherland is century old which started with Dutch coming India in

1605. However, the diplomatic relationship started with India gained its Independence in 1947.

The year 2017 marks the 70 year of Indo-Netherland Diplomatic relationship which both the country is

celebrating in full enthusiasm.

Despite the Eurozone crisis, Netherlands continues to exercise considerable influence, within EU and

NATO. Its importance is seen from the fact that it is among the few EU AAA rated countries by India.

-Netherland is the third largest source of FDI

Source: The Hindu (dated June 27th)

Tikdam:

This Question looks very tough, since first 2 statements are very factual and it is beyond the scope

of an average aspirant to remember these minute details.However it can be easily solved using the

Tikdam technique. Since any student who reads news-paper will be aware of the the fact that

biggest source of FSI in India is either Mauritius or Singapore.Thus statement 3 can

eliminated.Once we do that.We will automatically get the answer without even looking at first 2

statements.

QUESTION 90.

www.freeupscmaterials.org

freeupscmaterials.org

Page 65: QUESTION 1. Q.1 ) NISARGRUNA technology is related to Your ... · Exam Title : TS03 CA-JUNE 17 L2 Email : sandeepbiswas221@gmail.com Contact : 8250416813 Civilsdaily Email: hello@civilsdaily.com

Exam Title : TS03 CA-JUNE 17 L2

Email : [email protected]

Contact : 8250416813

Civilsdaily

Email: [email protected] 0

Q.90 ) Qatar has been accused of supporting terrorism by its neighbours and they have severed ties with

Qatar due to it. Which of the following countries have cut their ties with Qatar?

1. Saudi Arabia

2. India

3. USA

4. Yemen

Select the correct code

a) 1 and 2

b) 1 and 4

c) 2, 3 and 4

d) 1, 2, 3 and 4

Correct Answer: B

Your Answer: B

Explanation

On 9th June, nine countries cut off diplomatic ties with the tiny but rich state of Qatar. These nations

include three members of GCC (Gulf Cooperation Council), Bahrain, UAE and Saudi Arabia. The rest of the

countries are Egypt, the Maldives, Mauritania, Yemen and the U.N. backed government of Libya. Four

additional countries Djibouti, Jordan, Senegal and Chad have downgraded their ties with Qatar.

source:

http://www.huffingtonpost.com/entry/the-gcc-qatar-crisis-explained-all-you-need-to-know_us_5947b9ece4b024b7e0df4daf

Why in news?

Saudi Arabia, Egypt, the United Arab Emirates, Yemen, Libya, Bahrain and Maldives recently severed their

ties with Qatar, accusing it of supporting terrorism

QUESTION 91.

The Union Cabinet has decided the disinvestment process for the state-owned carrier Air India. Consider

the following statements about disinvestment:

www.freeupscmaterials.org

freeupscmaterials.org

Page 66: QUESTION 1. Q.1 ) NISARGRUNA technology is related to Your ... · Exam Title : TS03 CA-JUNE 17 L2 Email : sandeepbiswas221@gmail.com Contact : 8250416813 Civilsdaily Email: hello@civilsdaily.com

Exam Title : TS03 CA-JUNE 17 L2

Email : [email protected]

Contact : 8250416813

Civilsdaily

Email: [email protected] 0

1.Disinvestment is the process when Government sells its shares of a PSU, to private sector company /

individual.

2.The Department of Disinvestment and Public Assets Management falls under the Ministry of Heavy

Industries.

Which of the statements given above is/are correct?

a) 1 only

b) 2 only

c) Both 1 and 2

d) Neither 1 nor 2

Correct Answer: A

Your Answer: A

Explanation

-Statement 1 is correct.

-The department of divestment was formed in December 1999, which later was made the ministry of

disinvestment in September 2001. In May 2004, it was shifted to the ministry of finance as one of the

departments under it. Now, the department has been renamed as Department of Investment and Public

Asset Management (Dipam) which falls under the Ministry of Finance.

QUESTION 92.

The Free Movement Regime which permits people residing along the border to travel across the boundary

without visa restrictions is a unique arrangement between which countries?

a) India Bhutan

b) India Myanmar

c) India Afghanistan

d) India Srilanka

Correct Answer: B

Your Answer: B

Explanation

The India-Myanmar border has a unique arrangement in place called the Free Movement Regime (FMR).

FMR permits the tribes residing along the border to travel 16-km across the boundary without visa

restrictions.

www.freeupscmaterials.org

freeupscmaterials.org

Page 67: QUESTION 1. Q.1 ) NISARGRUNA technology is related to Your ... · Exam Title : TS03 CA-JUNE 17 L2 Email : sandeepbiswas221@gmail.com Contact : 8250416813 Civilsdaily Email: hello@civilsdaily.com

Exam Title : TS03 CA-JUNE 17 L2

Email : [email protected]

Contact : 8250416813

Civilsdaily

Email: [email protected] 0

Why in news?

The Union Home Ministry has constituted a committee to examine methods to curb the misuse of free

movement along the Myanmar border.

http://www.thehindu.com/news/national/panel-to-study-free-movement-along-myanmar-border/article18967743.ece

QUESTION 93.

Birdlife International has added three new sites in Goa to the list of Important Bird and Biodiversity Areas.

Consider the following statements about Birdlife International

1. It is an organ of United Nations that strives to conserve birds, their habitats and global biodiversity.

2. BirdLife International is the official Red List authority for birds, for the International Union for

Conservation of Nature.

Which of the statements given above is/are correct?

a) 1 only

b) 2 only

c) Both 1 and 2

d) Neither 1 nor 2

Correct Answer: B

Your Answer: B

Explanation

-BirdLife International, a conservation organisation, has recognized three new sites in Goa as hotspots for

protection. The sites have been added to their list of Important Bird and Biodiversity Areas.

-Goa earlier had four recognised biodiversity areas: Bhagwan Mahavir Wildlife Sanctuary and Mollem

National Park, Carambolim Wetlands, Cotigao Wildlife Sanctuary and Mhadei Wildlife Sanctuary.

-The list has now added Bondla Wildlife Sanctuary, Navelim Wetlands and Netravali Wildlife Sanctuary.

-BirdLife International (formerly the International Council for Bird Preservation) is a global partnership of

conservation organisations that strives to conserve birds, their habitats and global biodiversity, working with

people towards sustainability in the use of natural resources.

www.freeupscmaterials.org

freeupscmaterials.org

Page 68: QUESTION 1. Q.1 ) NISARGRUNA technology is related to Your ... · Exam Title : TS03 CA-JUNE 17 L2 Email : sandeepbiswas221@gmail.com Contact : 8250416813 Civilsdaily Email: hello@civilsdaily.com

Exam Title : TS03 CA-JUNE 17 L2

Email : [email protected]

Contact : 8250416813

Civilsdaily

Email: [email protected] 0

-It is the worlds largest partnership of conservation organisations, with over 120 partner organisations. It is

a not for profit, NGO not an organ of United Nations.

Statement 2 is correct.

http://www.thehindu.com/news/national/other-states/three-new-sites-recognised-as-biodiversity-hotspots-in-goa/article18966988.ece

QUESTION 94.

Which is the first state which formulated the Transgender Policy to enforce the constitutional rights of

transgenders ?

a) Kerala

b) Maharashtra

c) Karnataka

d) Haryana

Correct Answer: A

Your Answer: Unanswered

Explanation

Kerala was the first state which formulated the Transgender Policy to enforce the constitutional rights of

transgenders

Why in news?

Indias First Transgender Sports meet was organised by the Kerala State Sports Council at

Thiruvananthapuram.

QUESTION 95.

With which of the following Red Line Campaign is associated with

a) Controlling Naxalite problem

b) Curbing border infiltration

c) Controlling antibiotic resistance

d) None of the above

www.freeupscmaterials.org

freeupscmaterials.org

Page 69: QUESTION 1. Q.1 ) NISARGRUNA technology is related to Your ... · Exam Title : TS03 CA-JUNE 17 L2 Email : sandeepbiswas221@gmail.com Contact : 8250416813 Civilsdaily Email: hello@civilsdaily.com

Exam Title : TS03 CA-JUNE 17 L2

Email : [email protected]

Contact : 8250416813

Civilsdaily

Email: [email protected] 0

Correct Answer: C

Your Answer: Unanswered

Explanation

It is putting a red line on antibiotic packages to curb their over-the-counter sale.

Why in news?

WHO has revised antibiotics protocol to curb antibiotic resistance.

http://www.who.int/mediacentre/news/releases/2017/essential-medicines-list/en/

QUESTION 96.

Consider the following statements regarding Zika virus

1. Zika virus disease is caused by a virus transmitted primarily by Aedes mosquitoes.

2. Zika can be passed through sexual route from a person with Zika to his or her partners

3. No cases of Zika virus has been reported from India till now

Which of the statements given above is/are correct?

a) 3 only

b) 2,3 only

c) 1 and 2 only

d) 1,2 and 3

Correct Answer: C

Your Answer: Unanswered

Explanation

-Zika virus disease is caused by a virus transmitted primarily by Aedes mosquitoes.

-Zika can be passed through sex from a person with Zika to his or her partners.

-The WHO has placed India as a Category-2 country for Zika risk.

-A Category-2, the second highest on a four-point scale and that also includes 2015 Zika-hotspot Brazil,

indicates that the virus is being actively transmitted within the country.

-Until April, India was a Category-4 country.

-The three cases of Zika virus were reported from Bapunagar area in Ahmedabad, Gujarat.

www.freeupscmaterials.org

freeupscmaterials.org

Page 70: QUESTION 1. Q.1 ) NISARGRUNA technology is related to Your ... · Exam Title : TS03 CA-JUNE 17 L2 Email : sandeepbiswas221@gmail.com Contact : 8250416813 Civilsdaily Email: hello@civilsdaily.com

Exam Title : TS03 CA-JUNE 17 L2

Email : [email protected]

Contact : 8250416813

Civilsdaily

Email: [email protected] 0

Why in news?

Recently the United States sent out an advisory informing its citizens in India about the number of

confirmed Zika infections in India.

QUESTION 97.

Consider the following statements regarding International Labour Organization

1.India is a founder member of the International Labour Organization (ILO)

2.India recently ratified two key ILO conventions on child labour: the Minimum Age Convention and the

Worst Forms of Child Labour Convention

3 ILO Conventions are not legally binding

Which of the statements given above is/are correct?

a) 1 only

b) 2 only

c) 1 and 2 only

d) 1,2 and 3

Correct Answer: C

Your Answer: C

Explanation

-India is a founder member of the International Labour Organization (ILO), which came into existence in

1919. The ILO sets up International standards in the form of Conventions, Recommendations and Protocol.

-Conventions and Recommendations: ILO sets International labour standards in the form of conventions

and recommendations. They are legal instruments.

www.freeupscmaterials.org

freeupscmaterials.org

Page 71: QUESTION 1. Q.1 ) NISARGRUNA technology is related to Your ... · Exam Title : TS03 CA-JUNE 17 L2 Email : sandeepbiswas221@gmail.com Contact : 8250416813 Civilsdaily Email: hello@civilsdaily.com

Exam Title : TS03 CA-JUNE 17 L2

Email : [email protected]

Contact : 8250416813

Civilsdaily

Email: [email protected] 0

-Conventions are legally binding international treaties that may be ratified by member states while

recommendations serve as non-binding guidelines.

Why in news?

India has ratified two key ILO conventions on child labour: the Minimum Age Convention (No 138) and the

Worst Forms of Child Labour Convention (No 182).

http://www.thehindubusinessline.com/economy/policy/india-ratifies-two-key-ilo-conventions-on-child-labour/article9726319.ece

QUESTION 98.

Consider the following statements regarding National Mission on Cultural Mapping

1.It is a part of Ek Bharat Shreshtha Bharat initiative

2.This mission brings all the cultural activities under one web based umbrella for better results.

3.It opens a direct channel of communication of artists with the Government and peer to peer

communication among artists.

Which of the statements given above is/are correct?

a) 1 only

b) 2 only

c) 1 and 2 only

d) 1,2 and 3

Correct Answer: D

Your Answer: Unanswered

Explanation

National Mission on Cultural Mapping

Mission is a part of Ek Bharat Shreshtha Bharat initiative.

The mission aims at converting the vast and widespread cultural canvas of India into an objective cultural

map, designing a mechanism to fulfil the aspirations of the whole artist community of the nation and

preserving the rich cultural heritage of this country in the form of a cultural repository of artists and art

forms.

www.freeupscmaterials.org

freeupscmaterials.org

Page 72: QUESTION 1. Q.1 ) NISARGRUNA technology is related to Your ... · Exam Title : TS03 CA-JUNE 17 L2 Email : sandeepbiswas221@gmail.com Contact : 8250416813 Civilsdaily Email: hello@civilsdaily.com

Exam Title : TS03 CA-JUNE 17 L2

Email : [email protected]

Contact : 8250416813

Civilsdaily

Email: [email protected] 0

This Mission encompasses data mapping, demography building formalising the processes and bringing all

the cultural activities under one web based umbrella for better results.

The Mission also seeks to open a direct channel of communication of artists with the Government and peer

to peer communication among artists for talent honing and handholding of each other.

Why in news?

Ministry of Culture launched National Mission for Cultural Mapping

http://pib.nic.in/newsite/PrintRelease.aspx?relid=165655

QUESTION 99.

Which of the following was the first newspaper to be published in the country?

a) Hicky's Bengal Gazette

b) Samachar Darpan

c) Sambad Kaumadi

d) Sambad Pravakar

Correct Answer: A

Your Answer: A

Explanation

The first newspaper of the country, Hicky's Bengal Gazette was published from Kolkata in 1780. The paper

ceased publication on March 23, 1782. It was published just for two years.

Samachar Darpan was the first newspaper in Bengali language. It was published by Serampore Mission

press on May 23, 1818.

In 1821, a remarkable Bengali journal Sambad Kaumadi was published under the patronage of Rammohan

Roy.

Sambad Pravakar was the first Bengali daily newspaper published in 1839, patronized by Iswar Chandra

Gupta.

Why in news?

Governor of West Bengal, Shri Keshari Nath Tripathi released a commemorative volume on the two

hundred years of Bengali Newspapers.

http://www.business-standard.com/article/government-press-release/president-of-india-inaugurates-celebrations-to-commemorate-the-bicentenary-of-publication-117063001085_1.html

www.freeupscmaterials.org

freeupscmaterials.org

Page 73: QUESTION 1. Q.1 ) NISARGRUNA technology is related to Your ... · Exam Title : TS03 CA-JUNE 17 L2 Email : sandeepbiswas221@gmail.com Contact : 8250416813 Civilsdaily Email: hello@civilsdaily.com

Exam Title : TS03 CA-JUNE 17 L2

Email : [email protected]

Contact : 8250416813

Civilsdaily

Email: [email protected] 0

QUESTION 100.

Consider the following statements regarding Drought avoidance (DA)??

a) a) Ability of a plant species to complete its life cycle before the onset of drought.

b) Ability of plants to maintain relatively higher tissue water content, despite the water scarcity in the soil

c) Plant enduring low water content on its tissues through various adaptive traits.

d) plant species with adaptive features which enables plants to escape, avoid and tolerate drought stress

Correct Answer: B

Your Answer: Unanswered

Explanation

Plants have evolved to endure drought stress with morphological, physiological, and biochemical

adaptations:

1. Drought resistance (DR) is a broader term applied to plant species with adaptive features which enables

plants to escape, avoid and tolerate drought stress.

2. Drought escape (DE) is the ability of a plant species to complete its life cycle before the onset of drought.

Thereby, plants do not experience drought stress, as they are able to modulate their vegetative and

reproductive growth according to water availability. Eg rapid plant growth during wet season and little

growth during the dry season.

3. Drought avoidance (DA) involves the ability of plants to maintain relatively higher tissue water content,

despite the water scarcity in the soil (saving for a deficit day).

4. Drought tolerance (DT) involves the plant enduring low water content on its tissues through various

adaptive traits

Why in news?

It was in news because several part of India hit by drought.

http://www.thehindu.com/sci-tech/energy-and-environment/myriad-ways-in-which-plants-handle-drought-stress/article18713884.ece

www.freeupscmaterials.org

freeupscmaterials.org